I've given up Exam 6 NCLEX Questions

Pataasin ang iyong marka sa homework at exams ngayon gamit ang Quizwiz!

A patient is concerned about a terrorist attack because of living near the airport. Which response should the nurse make? 1. "Have you thought about moving?" 2. "That's silly to be so worried." 3. "What do you have in your home to help you in the event of a terrorist attack?" 4. "I would be concerned, too."

3. "What do you have in your home to help you in the event of a terrorist attack?"

A patient receives a chemical burn to the left side of the face and right wrist. Where does this patient need to be treated? 1. Outpatient ambulatory clinic 2. Emergency department 3. Burn center 4. Healthcare provider's office, and then at home

3. Burn center

A patient who has reacted poorly to general anesthesia in the past is scheduled for surgery to repair a rotator cuff tear. For which types of anesthesia should the nurse prepare educational materials for this patient? Select all that apply. 1. Spinal 2. Topical 3. Epidural 4. Nerve block 5. Local nerve infiltration

3. Epidural 4. Nerve block Rationale: Epidural blocks are local anesthetic agents injected into the epidural space, outside the dura mater of the spinal cord. It is indicated for surgeries of the shoulders. Nerve blocks are accomplished by injecting an anesthetic agent at the nerve trunk to produce a lack of sensation over a specific larger area, such as an extremity.

The nurse is supervising a nursing student who is delivering care to a client with a burn injury to the chest. Nitrofurazone is prescribed to be applied to the site of injury. The nurse should intervene if the student planned to implement which action to apply the medication? 1.Wash the burn site. 2.Apply the medication with a sterile gloved hand. 3.Apply saline-soaked dressings over the medication. 4.Apply 1/16-inch (1.5-mm) film directly to the burn sites.

3.Apply saline-soaked dressings over the medication. Rationale: Nitrofurazone is applied topically to the burn and has a broad spectrum of antibiotic activity. It is used in second- and third-degree burns when bacterial resistance to other agents is a potential problem. The burn site is washed before medication application. A film of 1/16 inch (1.5 mm) is applied directly to the burn using a sterile gloved hand. Saline-soaked dressings are not used with this medication because they will inactivate the medication's effect. In addition, wet dressings present the risk for infection, and infection is a primary concern with a client who is burned.

The community health nurse is working with disaster relief after a tornado. The nurse assists in finding safe housing for survivors, providing support to families, organizing counseling, and securing physical care when needed. Which level of prevention does the nurse exercise? 1.Primary level of prevention 2.Secondary level of prevention 3.Tertiary level of prevention 4.Quaternary level of prevention

3.Tertiary level of prevention

A patient reports nausea, vomiting, chills, and a headache after spending the weekend at a seaside resort. On which assessment should the nurse focus? 1. Sleeping pattern 2. Eating pattern 3. Travel issue including time zone changes 4. Time spent in the sun causing a sunburn

4. Time spent in the sun causing a sunburn

A patient being discharged is concerned about being overmedicated because of receiving a prescription for Demerol 50 mg by mouth when 10 mg of morphine was given through the intravenous catheter in the hospital. How should the nurse respond to this patient? 1. "Oral doses need to be higher than those given through an IV." 2. "The doctor is making sure that you do not have any pain once you go home." 3. "I will get the doctor so he can explain what is going on with your condition." 4. "All patients have more pain when they go home so the doctor is making sure you have enough medication."

1. "Oral doses need to be higher than those given through an IV." Rationale: Oral doses of analgesics are not equal to parenteral doses. The oral dose of an opioid such as morphine, codeine, or hydromorphone may be two to five times higher than the parenteral dose to achieve equivalent pain relief. This is what the nurse should explain to the patient.

A patient with multiple gunshot wounds has received 8 units of blood; however, the blood bank has run out of blood for the patient. For which blood type can the patient receive any type of blood? 1. AB 2. A 3. B 4. O

1. AB

A patient experiences blunt trauma to the abdomen after a motor vehicle crash. What should be assessed first? 1. Airway for patency 2. Abdomen for any abnormalities 3. Cervical spine for tenderness 4. Signs of neurological deficits

1. Airway for patency

A pediatric patient is admitted after ingesting a household cleaning solution. For which energy source should care be planned for this patient? 1. Chemical 2. Physical 3. Thermal 4. Mechanical

1. Chemical

A patient is brought to the emergency department with injuries sustained when a wall collapsed in the home. Which mechanism of injury most likely caused this patient's injuries? 1. Crushing 2. Shearing 3. Deceleration 4. Blast

1. Crushing

The nurse is assessing a patient's burns. What information should the nurse assess to classify this injury? Select all that apply. 1. Depth of the burn 2. Extent of burns on the body 3. Causative agent and duration of exposure 4. Location of burns on the body 5. Time that the burns occurred

1. Depth of the burn 2. Extent of burns on the body 3. Causative agent and duration of exposure 4. Location of burns on the body

The nurse is admitting the second patient in 2 days who is experiencing malaise, high fever, and headache. Both patients attended a local air show the previous weekend. What should the nurse do? 1. Discuss the need to contact public health authorities regarding the similarity of the cases. 2. Plan to place the patients in the same room. 3. Ask the patients for permission to talk with family members regarding other symptoms. 4. Assess the patients for what foods they ate while at the air show.

1. Discuss the need to contact public health authorities regarding the similarity of the cases.

After a serious highway accident involving 22 vehicles, ambulatory victims with minor injuries are transported home. For which reason were these victims triaged first? 1. Do the greatest good for the greatest number of people. 2. Remove them so they do not see the critically injured victims. 3. Secure the site in case the vehicles become overheated and ignite. 4. Improve traffic conditions to clear the accident site quickly.

1. Do the greatest good for the greatest number of people.

A derailed train caused a hazardous spill with noxious gasses released into the environment. Local and surrounding fire control and Hazmat teams are controlling the event. For which type of situation should the nurse prepare? 1. Emergency 2. Disaster 3. Human-generated accidental disaster 4. Intentional emergency

1. Emergency

The nurse applies Elase to a third-degree burn on a patient's left thigh and left forearm. Which type of wound debridement is the nurse using? 1. Enzymatic 2. Mechanical 3. Surgical 4. Topical

1. Enzymatic

The nurse is evaluating the adequacy of a burn-injured patient's nutritional intake. Which laboratory value indicates the need to adjust the patient's nutritional program? 1. Glycosuria 2. Creatine phosphokinase (CPK) 3. BUN levels 4. Hemoglobin

1. Glycosuria

A construction worker was admitted after falling from the roof of a building. Which energy source should the nurse use to plan this patient's care? 1. Gravitational 2. Mechanical 3. Physical 4. Electrical

1. Gravitational

A patient being prepared for surgery has been diagnosed with dehydration. Which laboratory values support the diagnosis for this patient? 1. Hemoglobin and hematocrit 2. Glucose 3. White blood cell count 4. Platelet count

1. Hemoglobin and hematocrit Rationale: An increase in hemoglobin and hematocrit levels would indicate dehydration.

An older patient is admitted after falling on the steps at home. Which component of trauma should the nurse consider when planning care for the patient? Select all that apply. 1. Host 2. Environment 3. Intention 4. Source 5. Transmission

1. Host 2. Environment 3. Intention

Emergency response personnel are wearing gowns, gloves, goggles, and masks. In which decontamination zone are these personnel most likely working? 1. Hot 2. Warm 3. Cold 4. Yellow

1. Hot

At the site of a collapsed building, some victims are walking away from the building, while others are lying on the ground around the perimeter. What should be done first when implementing reverse triage? 1. Identify one safe location for all ambulatory victims to be assessed and observed. 2. Systematically assess each victim lying on the ground and assign a color. 3. Enter the building to locate other victims who might not have been able to leave before the collapse. 4. Transfer the victims on the ground to stretchers and send them to the local emergency department.

1. Identify one safe location for all ambulatory victims to be assessed and observed.

A patient is having an epidural catheter inserted for pain control after surgery. What should the nurse realize is an advantage of using this method of pain medication for this patient? 1. Improved bowel activity 2. Faster wound healing 3. Earlier ambulation 4. Improved appetite

1. Improved bowel activity Rationale: This type of intraspinal anesthesia provides safe and effective pain relief for patients of all ages with less risk of adverse effects than general anesthesia.

A patient with severe burns to the torso and upper extremities has edema around the burned areas. How should the nurse describe the underlying cause for this assessment finding? 1. Inability of the damaged capillaries to maintain fluids in the cell walls 2. Reduced vascular permeability in the burned area 3. Decreased osmotic pressure in the burned tissue 4. Increased fluids in the extracellular compartment

1. Inability of the damaged capillaries to maintain fluids in the cell walls

An older patient is sunburned over much of the body. What self-care technique is MOST important for the nurse to emphasize to this patient? 1. Increasing fluid intake 2. Applying mild lotions 3. Taking mild analgesics 4. Maintaining warmth

1. Increasing fluid intake

A patient's postoperative wound has sanguineous drainage with a thick, reddish appearance. In which phase of healing is this patient's wound? 1. Inflammatory 2. Proliferative 3. Stationary 4. Remodeling

1. Inflammatory Rationale: The inflammatory phase begins with the surgical incision. Sanguineous drainage contains both serum and red blood cells and has a thick, reddish appearance.

A patient with a penetrating wound to the neck has dyspnea, cyanosis, and evidence of subcutaneous emphysema. What should the nurse expect to be completed first for this patient? 1. Intubate because of the severe wound. 2. Notify the next of kin. 3. Prepare for x-rays of the lumbar area to assess for fractures. 4. Administer a beta blocker to alleviate the sympathetic response.

1. Intubate because of the severe wound.

A patient with full thickness burns is being treated with high-volume intravenous fluids and has a urine output of 40 mL per hour. What does the nurse realize about this urine output? 1. It is normal for this patient. 2. It is evidence that the patient is dehydrated. 3. It is evidence that the patient is overhydrated. 4. It indicates pending renal failure.

1. It is normal for this patient.

A patient with third-degree burns is prescribed gastrointestinal medication. How should the nurse explain the primary action of this medication? 1. It prevents the formation of a Curling ulcer. 2. It treats a preexisting duodenal ulcer. 3. It ensures adequate peristalsis. 4. It has antiemetic properties.

1. It prevents the formation of a Curling ulcer.

A patient recovering from surgery, experiences a deep vein thrombosis (DVT). Which preoperative exercise should the nurse identify as being ineffective for this patient? 1. Leg exercises 2. Deep breathing and coughing 3. Use of incentive spirometry 4. Splinting when coughing

1. Leg exercises Rationale: Preoperative patient teaching includes leg exercises in order to reduce the onset of the complication deep vein thrombosis. The development of a DVT indicates teaching was ineffective.

A patient is admitted with possible head and spinal cord injuries sustained after falling from a ladder. Which diagnostic test should the nurse expect to be prescribed that will identify the type and extent of this patient's injuries? 1. Magnetic resonance imaging 2. Cervical spine x-rays 3. Spinal cord x-rays 4. Cerebral angiogram

1. Magnetic resonance imaging

An adult patient who has died from traumatic injuries is an organ donor. What should the nurse do when caring for this patient? 1. Maintain systolic blood pressure of 90 mmHg. 2. Keep oxygen saturation level at 75%. 3. Administer intravenous fluids to maintain a urine output of 25 mL per hour. 4. Perform external cardiac compressions to achieve a heart rate of 60 beats per minute.

1. Maintain systolic blood pressure of 90 mmHg.

A patient has sustained a partial-thickness burn of 28% of total body surface area (TBSA) and full-thickness burn of 30% or greater of TBSA. How should the nurse classify this burn injury? 1. Major 2. Moderate 3. Minor 4. Superficial

1. Major

Several victims of a suspected biologic attack are brought into the emergency department. Which type of personal protective equipment should be provided to these victims? 1. Mask 2. Gown 3. Gloves 4. Goggles

1. Mask

A small commuter plane crashed into a shopping mall, injuring approximately 500 people. Which classification of incident should the nurse use when determining the care needed for these victims? 1. Mass-casualty incident 2. Multiple-casualty incident 3. Accidental natural disaster 4. Intentional human-generated disaster

1. Mass-casualty incident

A patient with third-degree burns on the right arm is scheduled for passive range of motion to the extremity every 2 hours. What should the nurse do prior to this exercise session? 1. Medicate the patient for pain. 2. Empty the patient's indwelling catheter collection bag. 3. Change the patient's bed linens. 4. Change the dressing on the burn.

1. Medicate the patient for pain.

A patient is scheduled for extraction of a cataract. How should the nurse classify this patient's surgical procedure?

1. Minor elective Rationale: Surgical procedures are classified according to purpose, risk factor, technique, and urgency. Cataract extraction would be considered a minor elective surgery. Minor procedures carry minimal risk and minimal physical assault.

A patient with a burn injury is prescribed silver nitrate. Which nursing intervention should be included in the plan of care for this patient? Select all that apply. 1. Monitor daily weight. 2. Monitor serum sodium levels. 3. Prepare to change the dressings every 2 hours. 4. Report black skin discolorations. 5. Saturate the dressings every 2 hours with a 0.5% aqueous solution of silver nitrate.

1. Monitor daily weight. 2. Monitor serum sodium levels. 5. Saturate the dressings every 2 hours with a 0.5% aqueous solution of silver nitrate.

A school bus transporting approximately 60 students crashed on the side of the road. Which classification should the nurse use to describe the situation? 1. Multiple-casualty incident 2. Natural disaster 3. Human-generated disaster 4. Mass-casualty incident

1. Multiple-casualty incident Rationale: A multiple-casualty event does not exceed the capacity of local resources to provide needed medical care.

A patient received injuries during an earthquake. Which type of incident should the nurse identify as causing this patient's injuries? 1. Natural disaster 2. Human-generated disaster 3. Emergency 4. Multiple-casualty incident

1. Natural disaster

During the assessment of a postoperative patient's bowel sounds, the nurse auscultates absent sounds over all four abdominal quadrants. For which reason should the nurse identify interventions for this patient? 1. Paralytic ileus 2. Normal bowel function 3. The onset of flatus 4. The onset of stool

1. Paralytic ileus Rationale: A distended abdomen with absent bowel sounds may indicate paralytic ileus.

A local hospital has coordinated with the county emergency management team to plan and conduct disaster drills on a monthly basis. In which phase of disaster management planning is this hospital functioning? 1. Preparedness 2. Mitigation 3. Response 4. Evaluation

1. Preparedness

A patient is having a surgical procedure with conscious sedation. Which patient information should be provided to the healthcare provider before administering the anesthesia to the patient? Select all that apply. 1. The patient has a history of snoring. 2. The patient drank a cup of coffee two hours ago. 3. The patient wants to be asleep for the procedure. 4. The patient's father was hypertensive. 5. The patient has a history of gout.

1. The patient has a history of snoring. 2. The patient drank a cup of coffee two hours ago. Rationale: While all of this information leads to a greater understanding of the patient, that the patient snores and they are not NPO is essential information at this time.

A patient being treated with topical mafenide acetate for third-degree burns is demonstrating facial and neck edema. What does the nurse realize is the most likely reason? 1. The patient is developing hypersensitivity to the medication. 2. The patient is reacting positively to the medication. 3. The patient needs an increase in dosage of the medication. 4. The patient is not responding to the medication.

1. The patient is developing hypersensitivity to the medication.

A patient is brought to the emergency department with injuries sustained in a motor vehicle crash. What should the nurse consider as the cause of this patient's injuries? 1. Trauma 2. Not wearing a seat restraint 3. A drunk driver 4. Not paying attention while driving

1. Trauma

An older patient is receiving an NSAID for postoperative pain. What should the nurse assess in this patient? 1. Urine output 2. Blood pressure 3. Respiratory rate 4. Heart rate

1. Urine output Rationale: NSAIDs can be given safely to older patients, but they should be observed closely for side effects, particularly gastric and renal toxicity. The nurse should monitor the patient's urine output to determine renal function.

A patient receiving treatment for severe burns over more than half the body has an indwelling urinary catheter. When evaluating the patient's intake and output, what should the nurse take into consideration? 1. Urine output will be reduced in the first 24-48 hours and will then increase. 2. Urine output will be greatest in the first 24 hours after the burn injury. 3. Urine output will be reduced during the first 8 hours and will then increase as diuresis begins. 4. Urine output will be elevated due to the amount of intravenous fluids administered during the initial phases of treatment.

1. Urine output will be reduced in the first 24-48 hours and will then increase.

The nurse arrives at the site of a bombed building. When preparing to triage victims, in which zone should the nurse conduct decontamination? 1. Warm 2. Hot 3. Cold 4. Green

1. Warm

A patient is admitted to the emergency department with deep partial-thickness burns over 35% of the body. What IV solution will be started initially? 1. Warmed lactated Ringer's solution 2. Dextrose 5% with saline solution 3. Dextrose 5% with water 4. Normal saline solution

1. Warmed lactated Ringer's solution

A patient receives an electrical burn. What assessment question should the nurse ask to determine the severity of the burn injury? Select all that apply. 1. What type of current was involved? 2. How long was the patient in contact with the current? 3. How much voltage was involved? 4. Where was the patient when the burn occurred? 5. What was the point of contact with the current?

1. What type of current was involved? 2. How long was the patient in contact with the current? 3. How much voltage was involved?

A patient is beginning the acute phase of burn treatment. What should the nurse anticipate as priorities for this patient's care? Select all that apply. 1. Wound care 2. Nutritional therapy 3. Infection control 4. Graft procedures 5. Home maintenance management

1. Wound care 2. Nutritional therapy 3. Infection control

The nurse is the first responder at the scene of a 6-car crash on a highway. Which victim should the nurse attend to first? 1.A victim experiencing dyspnea 2.A victim experiencing confusion 3.A victim experiencing tachycardia 4.A victim experiencing intense pain

1.A victim experiencing dyspnea

A burn-injured client is receiving treatments of topical mafenide acetate to the site of injury. The nurse should monitor the client for which systemic effect that can occur from the use of this medication? 1.Acidosis 2.Alkalosis 3.Hypotension 4.Hypertension

1.Acidosis Rationale: Mafenide acetate is a carbonic anhydrase inhibitor and can suppress renal excretion of acid, thereby causing acidosis. Clients receiving this treatment should be monitored for acidosis, and if the acidosis becomes severe, the medication should be discontinued for 1 to 2 days. An elevated blood pressure may be expected in the client with pain. Alkalosis and hypotension are not associated with the use of this medication.

A burn client is receiving treatments of topical mafenide acetate to the site of injury. The nurse monitors the client, knowing that which finding indicates that a systemic effect has occurred? 1.Hyperventilation 2.Elevated blood pressure 3.Local rash at the burn site 4.Local pain at the burn site

1.Hyperventilation Rationale: Mafenide acetate is a carbonic anhydrase inhibitor and can suppress renal excretion of acid, thereby causing acidosis. Clients receiving this treatment should be monitored for signs of an acid-base imbalance (hyperventilation). If this occurs, the medication will probably be discontinued for 1 to 2 days. Options 3 and 4 describe local rather than systemic effects. An elevated blood pressure may be expected from the pain that occurs with a burn injury.

The emergency department nurse is caring for an adult client who is a victim of family violence. Which priority information should be included in the discharge instructions? 1.Information regarding shelters 2.Instructions regarding calling the police 3.Instructions regarding self-defense classes 4.Explaining the importance of leaving the violent situation

1.Information regarding shelters

The nurse has developed a nursing care plan for a client with a burn injury to implement during the emergent phase. Which priority intervention should the nurse include in the plan of care? 1.Monitor vital signs every 4 hours. 2.Monitor mental status every hour. 3.Monitor intake and output every shift. 4.Obtain and record weight every other day.

1.Monitor vital signs every 4 hours. Rationale: In a client with ineffective tissue perfusion related to a circumferential burn injury, peripheral pulses should be assessed every hour for 72 hours. The affected extremities should be elevated, and the primary health care provider (PHCP) should be notified of any changes in pulses, capillary refill, or pain sensation. Pressure dressings and wraps should not be applied around the circumferential burn because they could cause a further alteration in peripheral circulation.

A series of thunderstorms ripped through a small town, causing massive power outages. The community nurse is reviewing the list of community members who might be adversely affected by the loss of power. For which residents should the nurse provide immediate care and support? Select all that apply. 1. 16-year-old male with asthma 2. 75-year-old male with sleep apnea 3. 72-year-old male with a pacemaker 4. 45-year-old female with multiple sclerosis 5. 86-year-old female who requires continuous oxygen 2 liters nasal cannula

2. 75-year-old male with sleep apnea 5. 86-year-old female who requires continuous oxygen 2 liters nasal cannula

A community hospital has been notified of a plane crash that occurred during a local air show. Which information will the hospital leadership need to prepare for the crash victims? Select all that apply. 1. Number of families who live in the community 2. Number of available beds in the hospital 3. Telephone list of staff to be called in to work immediately 4. Amount of intravenous fluids and emergency medications 5. List of current patients who can be immediately discharged

2. Number of available beds in the hospital 3. Telephone list of staff to be called in to work immediately 4. Amount of intravenous fluids and emergency medications 5. List of current patients who can be immediately discharged

A client arrives at the health care clinic and tells the nurse that he was just bitten by a tick and would like to be tested for Lyme disease. The client reports that he removed the tick and flushed it down the toilet. The nurse should take which nursing action? 1.Refer the client for a blood test immediately. 2.Ask the client about the size and color of the tick. 3.Tell the client to return to the clinic in 4 to 6 weeks. 4.Inform the client that the tick is needed to perform a test.

3.Tell the client to return to the clinic in 4 to 6 weeks.

During a disaster preparedness presentation, the nurse learns about the harmful effects of a dirty bomb. Which treatment should the nurse realize will preserve life in the event of this disaster? 1. Support for the patient who will develop radiation sickness 2. Heart-lung transplant 3. Liver transplant 4. Bone marrow transplant

4. Bone marrow transplant

A client arriving at the emergency department has experienced frostbite to the right hand. Which finding would the nurse note on assessment of the client's hand? 1.A pink, edematous hand 2.Fiery red skin with edema in the nail beds 3.Black fingertips surrounded by an erythematous rash 4.A white color to the skin, which is insensitive to touch

4.A white color to the skin, which is insensitive to touch

A client exhibits a purplish bruise to the skin after a fall. The nurse would document this finding in the health record most accurately using which term? 1.Purpura 2.Petechiae 3.Erythema 4.Ecchymosis

4.Ecchymosis

A patient who is being admitted for surgery asks why information is being collected about the use of herbal and natural supplements. How should the nurse respond to this patient? 1. "Herbal supplements may interact with anesthetic agents." 2. "Herbal remedies may cause pain relievers to be ineffective." 3. "The physician is in charge of medications." 4. "There is no need to take these preparations."

1. "Herbal supplements may interact with anesthetic agents." Rationale: The use of herbal supplements must be documented prior to surgery. It is possible for these elements to interact with anesthetic agents.

A victim of a plane crash exits the plane and tells emergency personnel that a family member is inside and unable to walk. Which action should be taken? 1. Assist with clearing all the victims who are ambulatory first, then locate the trapped victim. 2. Enter the plane to locate the trapped victim. 3. Tell the victim that the family member will be fine and will be removed from the plane as soon as possible. 4. Tell the victim to go back inside and try to get the family member to walk.

1. Assist with clearing all the victims who are ambulatory first, then locate the trapped victim.

The nurse is assisting a postoperative patient in using an incentive spirometer. Which postoperative complication is this nurse attempting to avoid? 1. Atelectasis 2. Deep vein thrombosis 3. Hemorrhage 4. Pulmonary embolism

1. Atelectasis Rationale: Promoting lung expansion and systemic oxygenation of tissues is a goal in preventing atelectasis. Nursing care includes assisting with incentive spirometry.

A patient comes into the emergency department with leg pain after falling on ice. For which classification of injuries should the nurse expect to plan care for this patient? 1. Class 3 minor 2. Class 1 minor 3. Class 1 penetrating 4. Class 3 penetrating

1. Class 3 minor

A patient is being transferred from the operating room to the recovery room. In which phase of the surgical process will the nurse in the recovery room provide care? 1. Postoperative 2. Preoperative 3. Intraoperative 4. Restorative

1. Postoperative Rationale: The postoperative phase begins when the patient is admitted to the recovery room and ends with the patient's recovery from the surgical intervention.

The nurse identifies the problem of impaired physical mobility in a patient hospitalized with traumatic injuries. Which intervention should the nurse identify as appropriate for this patient? 1. Provide active range-of-motion exercises to affected extremities every 8 hours. 2. Turn and reposition every hour. 3. Remove anti-embolic stockings for 3 hours every shift. 4. Administer tetanus toxoid.

1. Provide active range-of-motion exercises to affected extremities every 8 hours.

A patient is brought to the emergency department with gunshot wounds to the abdomen and lower extremities. To protect the chain of evidence for these injuries, what should the nurse do? 1. Remove the patient's clothing and place in a breathable bag. 2. Cover the patient's hands with plastic bags. 3. Cut off the patient's clothing and bathe the skin and wounds as soon as possible. 4. Place clothing and other patient items on a bedside table and have a nursing assistant remove them when possible.

1. Remove the patient's clothing and place in a breathable bag.

Following surgical debridement, a patient with third-degree burns does not bleed. What should this outcome suggest to the nurse? 1. The procedure will need to be repeated. 2. The patient will no longer need this procedure. 3. The patient will need to be premedicated prior to the next procedure. 4. The patient should have an escharotomy instead.

1. The procedure will need to be repeated.

he emergency department (ED) nurse receives a telephone call and is informed that a tornado has hit a local residential area and that numerous casualties have occurred. The victims will be brought to the ED. The nurse should take which initial action? 1.Prepare the triage rooms. 2.Activate the emergency response plan. 3.Obtain additional supplies from the central supply department. 4.Obtain additional nursing staff to assist in treating the casualties.

2.Activate the emergency response plan.

A client is diagnosed with a full-thickness burn. What should the nurse anticipate will be used for final coverage of the client's burn wound? 1.Biobrane 2.Autograft 3.Xenograft 4.Homograft

2.Autograft Rationale: A full-thickness burn will require terminal coverage with an autograft-the client's own skin. Biobrane is porcine collagen bonded to a silicone membrane, which is temporary and lasts anywhere from 10 to 21 days. Homografts (cadaveric skin) and xenografts (pigskin) provide temporary coverage of the wound by acting as a dressing for up to 3 weeks before rejecting.

A client with a burn injury is applying mafenide acetate cream to the wound. The client calls the primary health care provider's (PHCP's) office and tells the nurse that the medication is uncomfortable and is causing a burning sensation. The nurse should instruct the client to take which action? 1.Discontinue the medication. 2.Continue with the treatment, as this is expected. 3.Apply a thinner film than prescribed to the burn site. 4.Come to the office to see the PHCP immediately.

2.Continue with the treatment, as this is expected. Rationale: Topical mafenide acetate is used to treat partial- and full-thickness burns. It is bacteriostatic for both gram-negative and gram-positive organisms present in avascular tissues. The client should be warned that the medication will cause local discomfort and burning. The nurse does not instruct a client to alter a medication prescription (options 1 and 3). It is not necessary that the client see the PHCP at this time.

The emergency department nurse is performing an assessment on a child suspected of being sexually abused. Which assessment data obtained by the nurse most likely support this suspicion? 1.Poor hygiene 2.Difficulty walking 3.Fear of the parents 4.Bald spots on the scalp

2.Difficulty walking

The nurse should plan to take which action nextafter assessing a homeless pediatric client who is a victim of abuse? 1.Ask the mother who abused the child. 2.Report signs of abuse and document it. 3.Find out where the child sleeps at night. 4.Ask the child if he or she is scared of her mother or anyone else.

2.Report signs of abuse and document it.

The nurse observes that a client with a potential for violence is agitated, pacing up and down the hallway, and making aggressive and belligerent gestures at other clients. Which statement would be most appropriate to make to this client? 1."You need to stop that behavior now." 2."You will need to be placed in seclusion." 3."You seem restless; tell me what is happening." 4."You will need to be restrained if you do not change your behavior."

3."You seem restless; tell me what is happening."

The nurse is caring for a female client in the emergency department who presents with a complaint of fatigue and shortness of breath. Which physical assessment findings, if noted by the nurse, warrant a need for follow-up? 1.Reddened sclera of the eyes 2.Dry flaking noted on the scalp 3.A reddish-purple mark on the neck 4.A scaly rash noted on the elbows and knees

3.A reddish-purple mark on the neck

The nurse from a medical unit is called to assist with care for clients coming into the hospital emergency department during an external disaster. Using principles of triage during a disaster, the nurse should attend to the client with which problem first? 1.Fractured tibia 2.Penetrating abdominal injury 3.Bright red bleeding from a neck wound 4.Open massive head injury in deep coma

3.Bright red bleeding from a neck wound

The clinic nurse reads the chart of a client just seen by the primary health care provider (PHCP) and notes that the PHCP has documented that the client has stage III Lyme disease. Which clinical manifestation should the nurse expect to note in this client? 1.Generalized skin rash 2.Cardiac dysrhythmia 3.Complaints of joint pain 4.Paralysis of the affected extremity

3.Complaints of joint pain

The nurse is caring for a client with full-thickness circumferential burns of the entire trunk of the body who is on a mechanical ventilator. Which finding suggests that an escharotomy may be necessary? 1.Pallor of all extremities 2.Pulse oximetry reading of 93% 3.Peripheral pulses are diminished 4.High pressure alarm keeps sounding on the ventilator

4.High pressure alarm keeps sounding on the ventilator Rationale: A client with a circumferential burn of the entire trunk likely will be on a ventilator because of the potential for breathing to be affected by this injury. The high pressure alarm will sound on the ventilator when there is any kind of obstruction. If the chest cannot expand due to restriction by eschar and increasing edema, this results in obstruction.

The nurse is conducting a session about the principles of first aid and is discussing the interventions for a snakebite to an extremity. The nurse should inform those attending the session that the first-priority intervention in the event of this occurrence is which action? 1.Immobilize the affected extremity. 2.Remove jewelry and constricting clothing from the victim. 3.Place the extremity in a position so that it is below the level of the heart. 4.Move the victim to a safe area away from the snake and encourage the victim to rest.

4.Move the victim to a safe area away from the snake and encourage the victim to rest.

The family of a patient with third-degree burns wants to know why "the scabs are being cut off" the patient's leg. What is the most appropriate response by the nurse? 1. "The scabs are really old burned tissue and need to be removed to promote healing." 2. "I'll ask the doctor to come and talk with you about the treatment plan." 3. "The patient asked for the scabs to be removed." 4. "The scabs are removed to check for blood flow to the burned area."

1. "The scabs are really old burned tissue and need to be removed to promote healing."

An older patient asks what can be done to prepare for a disaster in the community. How should the nurse respond? 1. "There is not really much we can do to be prepared." 2. "Plan to evacuate your home at a moment's notice." 3. "Make sure all your important papers, health information, medication information, and next-of-kin information is in one place." 4. "Make sure you can call your family to come and pick you up if this happens."

3. "Make sure all your important papers, health information, medication information, and next-of-kin information is in one place."

A victim of a fire in a clothing shop is complaining of headache and dizziness and has a potentially dangerous heart rhythm. What should the nurse expect to be prescribed for this patient? Select all that apply. 1. Treatment with prednisone 2. Treatment with vancomycin 3. Treatment with hydroxocobalamin 4. Hyperbaric oxygen therapy 5. Pacemaker insertion

3. Treatment with hydroxocobalamin 4. Hyperbaric oxygen therapy

The nurse is reviewing the patient's current medications as a part of preparation for an elective surgery. What information should the nurse reinforce with the patient? 1. "Continue to take your regular prescribed dose of warfarin (Coumadin) until told otherwise." 2. "You may take your regular herbal supplements up until the day before surgery." 3. "Discontinue your antihypertensive medications two days prior to surgery." 4. "Stop taking your daily aspirin at least three days prior to surgery."

4. "Stop taking your daily aspirin at least three days prior to surgery." Rationale: Anticoagulant medications should be discontinued prior to surgery to prevent excessive blood loss during surgery. These include aspirin.

A patient is brought into the emergency department complaining of blindness after an explosion at an atomic power plant. What should the nurse explain to this patient? 1. "The effects of the blast will disappear with time." 2. "I'm afraid the blast has caused permanent blindness." 3. "I will contact Social Services so that resources can be identified to help you with your disability." 4. "There is a variety of resources available for those who are unable to see."

1. "The effects of the blast will disappear with time."

A patient gets struck by lightning during a thunderstorm on a golf course. What is the first action that should occur in this patient's care? 1. Check breathing and circulation. 2. Look for entrance and exit wounds. 3. Cover the victim to prevent heat loss. 4. Move the victim indoors to a dry place.

1. Check breathing and circulation.

A patient is brought into the emergency department with chemical burns. What should the nurse do to help this patient? 1. Check to see if all clothing has been removed and begin flushing the patient's skin with water. 2. Begin flushing the patient's clothes and skin with warm water. 3. Do not remove any jewelry. 4. Keep the patient's contact lenses in place and flush only with warm water.

1. Check to see if all clothing has been removed and begin flushing the patient's skin with water.

At the site of a building collapse, ambulances are nearby and a shelter has been set up with carts, chairs, tables, and refreshments for the victims. Within which decontamination zone are the victims being assessed? 1. Cold 2. Warm 3. Control 4. Hot

1. Cold

A patient has sustained a burn injury. Which nursing intervention is of the highest priority at this time? 1. Determine the type of burn injury 2. Determine the types of home remedies attempted prior to the patient's coming to the hospital 3. Assess past medical history 4. Measure body weight

1. Determine the type of burn injury

An older patient is recovering from a surgical procedure. What should the nurse do to ensure the patient is comfortable? 1. Provide warm blankets. 2. Limit movement to once every eight hours. 3. Explain all activities using a loud voice. 4. Limit fluids.

1. Provide warm blankets. Rationale: The older patient may need extra blankets for warmth. This is what the nurse should do to ensure for the patient's comfort.

A patient with third-degree burns has a reduction in the serum potassium level. To which event is this laboratory value related? 1. Resolution of burn shock 2. Onset of burn shock 3. Onset of renal failure 4. Onset of liver failure

1. Resolution of burn shock

A patient with injuries from a motor vehicle crash develops hypotension and severe jugular distension with a tracheal deviation. What should the nurse suspect is occurring in this patient? 1. Tension pneumothorax 2. Hemorrhage 3. Compensatory shock 4. Hypovolemic shock

1. Tension pneumothorax

A patient receives a chemical burn from contact with lye. Which information should guide the planning of care for this patient? Select all that apply. 1. This is an alkali burn. 2. This type of burn tends to be deeper. 3. This is an acid burn. 4. This type of burn is easier to neutralize. 5. This type of burn tends to be more superficial.

1. This is an alkali burn. 2. This type of burn tends to be deeper.

An individual with injuries sustained in a natural gas explosion is unable to respond to questions and keeps repeating "I can't hear." For what should the nurse assess this patient? 1. Tympanic membrane rupture 2. Air embolism 3. Oxygen saturation level 4. Confusion

1. Tympanic membrane rupture

A client who has had abdominal surgery complains of feeling as though "something gave way" in the incisional site. The nurse removes the dressing and notes the presence of a loop of bowel protruding through the incision. Which interventions should the nurse take? Select all that apply. 1.Contact the surgeon. 2.Instruct the client to remain quiet. 3.Prepare the client for wound closure. 4.Document the findings and actions taken. 5.Place a sterile saline dressing and ice packs over the wound. 6.Place the client in a supine position without a pillow under the head.

1.Contact the surgeon. 2.Instruct the client to remain quiet. 3.Prepare the client for wound closure. 4.Document the findings and actions taken.

The emergency department nurse is caring for a client who has sustained chemical burns to the esophagus after ingestion of lye. The nurse reviews the primary health care provider's prescriptions and should plan to question which prescription? 1.Gastric lavage 2.Intravenous (IV) fluid therapy 3.Nothing by mouth (NPO) status 4.Preparation for laboratory studies

1.Gastric lavage Rationale: The client who has sustained chemical burns to the esophagus is placed on NPO status, is given IV fluids for replacement and treatment of possible shock, and is prepared for esophagoscopy and barium swallow to determine the extent of damage. Laboratory studies also may be prescribed. A nasogastric tube may be inserted, but gastric lavage and emesis are avoided to prevent further erosion of the mucosa by the irritating substances that these treatments involve.

An unusually high number of patients are coming into the emergency department with complaints of nausea, vomiting, and severe headache. What should be done with this information? 1. No further action is needed at this time. 2. Contact the Infection Control Department and Laboratory Medicine. 3. Close the emergency department. 4. Call for more staffing to handle all the patients.

2. Contact the Infection Control Department and Laboratory Medicine.

A patient who sustained full-thickness burns six months ago is being evaluated for ongoing care needs. Which finding indicates that the patient's wounds are in the remodeling phase of healing? Select all that apply. 1. Scars fading in color 2. Hypertrophic scarring 3. Scattered areas of keloids 4. Granulation tissue is present 5. Burned tissue resembles neighboring tissue

2. Hypertrophic scarring 3. Scattered areas of keloids

A patient seeks medical attention for a burn that appears moist with blisters. For which type of burn should the nurse plan care for this patient? 1. Superficial 2. Superficial partial-thickness 3. Deep partial-thickness 4. Full thickness

2. Superficial partial-thickness

The nurse is caring for a client who sustained a thermal burn caused by the inhalation of steam 24 hours ago. The nurse determines that the priority nursing action is to assess which item? 1.Pain level 2.Lung sounds 3.Ability to swallow 4.Laboratory results

2.Lung sounds Rationale: The priority nursing action would be to assess lung sounds. Thermal burns to the lower airways can occur with the inhalation of steam or explosive gases or with the aspiration of scalding liquids. Thermal burns to the upper airways are more common and cause erythema and edema of the airways and mucosal blisters or ulcerations. The mucosal edema can lead to upper airway obstruction, particularly during the first 24 to 48 hours after a burn injury.

The nurse manager is discussing the facility protocol in the event of a tornado with the staff. Which instructions should the nurse manager include in the discussion? Select all that apply. 1.Open doors to client rooms. 2.Move beds away from windows. 3.Close window shades and curtains. 4.Place blankets over clients who are confined to bed. 5.Relocate ambulatory clients from the hallways back into their rooms.

2.Move beds away from windows. 3.Close window shades and curtains. 4.Place blankets over clients who are confined to bed.

The nurse assesses a client's surgical incision for signs of infection. Which finding by the nurse would be interpreted as a normal finding at the surgical site? 1.Red, hard skin 2.Serous drainage 3.Purulent drainage 4.Warm, tender skin

2.Serous drainage

A 15-year-old pregnant, unwed client tells the nurse, "My life was unbearable before I met Bobby. My mother beats me every day, and my dad has sexually abused me since I was 10 years old!" Which response is appropriate for the nurse to make? 1."Why didn't you just report your parents for this abuse?" 2."What are you saying? Your parents abused you, so you got pregnant?" 3."It seems that you needed Bobby's help to separate from your family." 4."Sounds like you decided to have a baby so you'd have someone for yourself."

3."It seems that you needed Bobby's help to separate from your family."

The nurse is providing care for a client who sustained burns over 30% of the body from a fire. On assessment, the nurse notes that the client is edematous in both burned and unburned body areas. The client's wife asks why her husband "looks so swollen." What is the nurse's best response? 1."Constricted blood vessels have caused a loss of protein in the blood." 2."Leaking blood vessels have led to increased protein amounts in the blood." 3."Leaking blood vessels have led to decreased protein amounts in the blood." 4."Constricted blood vessels have led to increased protein amounts in the blood."

3."Leaking blood vessels have led to decreased protein amounts in the blood." Rationale: In extensive burn injuries (greater than 25% of total body surface area), the edema occurs in both burned and unburned areas as a result of the increase in capillary permeability and hypoproteinemia. Edema also may be caused by the volume and oncotic pressure effects of the large fluid resuscitation volumes required.

The nurse is the first responder at the scene of a train accident. Which victim should the nurse attend to first? 1.A middle-aged man with 1 foot trapped under the wreckage 2.A crying teenager who is holding pressure on an arm laceration 3.A young woman who appears dazed and confused and is shivering 4.A screaming middle-aged woman looking frantically for her husband

3.A young woman who appears dazed and confused and is shivering

The nurse is performing an assessment on a client admitted to the nursing unit who has sustained an extensive burn injury involving 45% of total body surface area. When planning for fluid resuscitation, the nurse should consider that fluid shifting to the interstitial spaces is greatest during which time period? 1.Immediately after the injury 2.Within 12 hours after the injury 3.Between 18 and 24 hours after the injury 4.Between 42 and 72 hours after the injury

3.Between 18 and 24 hours after the injury Rationale: The maximum amount of edema in a client with a burn injury is seen between 18 and 24 hours after the injury. With adequate fluid resuscitation, the transmembrane potential is restored to normal within 24 to 36 hours after the burn. The remaining options are incorrect.

A client comes to the emergency department after an assault and is extremely agitated, trembling, and hyperventilating. What is the priority nursing action for this client? 1.Begin to teach relaxation techniques. 2.Encourage the client to discuss the assault. 3.Remain with the client until the anxiety decreases. 4.Place the client in a quiet room alone to decrease stimulation.

3.Remain with the client until the anxiety decreases.

The emergency department nurse is caring for a client who has been identified as a victim of physical abuse. In planning care for the client, which is the priority nursing action? 1.Adhering to the mandatory abuse reporting laws 2.Notifying the case worker of the family situation 3.Removing the client from any immediate danger 4.Obtaining treatment for the abusing family member

3.Removing the client from any immediate danger

The emergency department nurse is caring for a client who has been identified as a victim of physical abuse. In planning care for the client, which is the priority nursing action? 1.Adhering to the mandatory abuse-reporting laws 2.Notifying the caseworker of the family situation 3.Removing the client from any immediate danger 4.Obtaining treatment for the abusing family member

3.Removing the client from any immediate danger

A female arrives at the site of a disaster, hysterically crying because of being in the building that collapsed just minutes earlier. What should the nurse do with this patient? 1. Advise to go home and be with family. 2. Triage the patient and transport to the hospital. 3. Have a nurse talk with the patient. 4. Ask psychiatric service personnel to talk with this patient.

4. Ask psychiatric service personnel to talk with this patient.

Which statement by the nurse indicates a need for further teaching concerning family violence? 1."Abusers use fear and intimidation." 2."Abusers usually have poor self-esteem." 3."Abusers often are jealous or self-centered." 4."Abusers are more often from low-income families."

4."Abusers are more often from low-income families."

A 10-year-old referred for evaluation after drawing sexually explicit scenes says to the psychiatric nurse, "I just felt like it." Which response by the nurse is focused on assessing for abuse-related symptoms? 1."Well, a picture paints a thousand words." 2."You just felt like destroying your textbooks?" 3."Your parents and teachers are very concerned about your drawings." 4."I am concerned about you. Are you now or have you ever been abused?"

4."I am concerned about you. Are you now or have you ever been abused?"

The camp nurse asks the children preparing to swim in the lake if they have applied sunscreen. The nurse reminds the children that chemical sunscreens are most effective when applied at which times? 1.Immediately before swimming 2.5 minutes before exposure to the sun 3.Immediately before exposure to the sun 4.At least 30 minutes before exposure to the sun

4.At least 30 minutes before exposure to the sun Rationale: Sunscreens are most effective when applied at least 30 minutes before exposure to the sun so that they can penetrate the skin. All sunscreens should be reapplied after swimming or sweating.

A client sustains a burn injury to the entire right and left arms, the right leg, and the anterior thorax. According to the rule of nines, the nurse would assess that this injury constitutes which body percentage? Fill in the blank.

Arms x2 = 18% anterior thorax = 18% Leg = 18% Total = 56%

After complaining of discomfort from a surgical procedure, the patient voices fear of addiction with taking analgesics as prescribed. What information should be provided to the patient regarding these concerns? Select all that apply. 1. "Addiction to opioid analgesics is rare when used for short-term postoperative pain management." 2. "Psychological tolerance is not commonly experienced by patients who take narcotic analgesics during the postoperative experience." 3. "Pain tolerance and the need for opioid analgesics are individualized." 4. "Patients should be screened for addiction potential prior to being given narcotics." 5. "I'll turn the TV on to help distract you from your pain."

1. "Addiction to opioid analgesics is rare when used for short-term postoperative pain management." 2. "Psychological tolerance is not commonly experienced by patients who take narcotic analgesics during the postoperative experience." 3. "Pain tolerance and the need for opioid analgesics are individualized." Rationale: The use of opioid analgesics during the postoperative period is rarely associated with physical & psychological dependency concerns. The pain management needs of patients will vary and should be managed individually

A patient believes that scheduled surgery is minor since it will be done as an outpatient. How should the nurse respond to this patient? 1. "Every surgical procedure is serious, and I will make sure you have information to have a successful recovery." 2. "You are right." 3. "If it were more serious, you would be admitted to the hospital." 4. "Your insurance plan does not cover inpatient surgical procedures. That's why your surgery is being done as an outpatient."

1. "Every surgical procedure is serious, and I will make sure you have information to have a successful recovery." Rationale: The complexity of the surgery and recovery and the expected level of care needed on completion of the surgery are the major differences between inpatient and outpatient surgical procedures. The outpatient surgical patient and family must cope with the additional stress of needing to learn a great deal of information in a short span of time. The nurse should explain that every surgical procedure is serious and that the patient will be given information to have a successful recovery.

An older patient who is preparing for surgery wants to wear glasses and keep a hearing aid in place until receiving anesthesia. Which nursing response demonstrates accurate therapeutic communication? 1. "I will contact the surgery department to discuss your requests." 2. "You cannot keep those in." 3. "The policies in the surgery unit will not allow it." 4. "Certainly, you can keep them for that time."

1. "I will contact the surgery department to discuss your requests." Rationale: To decrease confusion and assist in communication, hearing aids and glasses should be used when appropriate and possible. The nurse will need to check with the surgical department first before granting the patient's wish.

A patient recovering from a traumatic gunshot wound asks when discharge will occur. Which response should the nurse make? 1. "Right now there is no way of knowing how long you will be hospitalized." 2. "I would say in a few weeks." 3. "Probably never." 4. "As soon as the wound heals, you can return to work."

1. "Right now there is no way of knowing how long you will be hospitalized."

A patient recovering from a motor vehicle accident asks how so many injuries occurred when the other car "barely" made contact. How should the nurse respond? 1. "The car that hit you transferred a large amount of energy to your body, causing these injuries." 2. "You have other health problems that make the injuries worse." 3. "The driver of the other car intended to hit you." 4. "Because you are older, your injuries will be worse."

1. "The car that hit you transferred a large amount of energy to your body, causing these injuries."

The spouse of a patient with a severe head injury believes the patient is going to recover because of making movements without stimulation. What should the nurse explain to the spouse? 1. "With head injuries, the patient can continue to make movements, but these are reflexes that do not reflect brain function." 2. "Your spouse will likely recover in time." 3. "As long as oxygen gets to the brain, the patient will recover." 4. "Those movements indicate that the brain is dead."

1. "With head injuries, the patient can continue to make movements, but these are reflexes that do not reflect brain function."

A patient having a hernia repair as an outpatient asks why hospitalization afterward is not required. Which response is appropriate for the nurse to make? Select all that apply. 1. "You have less risk of getting an infection at home." 2. "You will probably be more comfortable in your own bed at home." 3. "It is cheaper for the insurance company if you go home today." 4. "The government won't let you stay." 5. "If you ask the healthcare provider, the hospital will probably let you stay."

1. "You have less risk of getting an infection at home." 2. "You will probably be more comfortable in your own bed at home." Rationale: Advantages of outpatient surgery is reduced risk of healthcare-associated infection and outpatient surgery is less physiologic stress.

A patient is scheduled for surgery to graft a burn injury on the arm. Which statement should the nurse include when instructing the patient about the procedure? 1. "You will begin to perform exercises to promote flexibility and reduce contractures after 5 days." 2. "You will need to report any itching, as it might signal infection." 3. "Performing the procedure near the end of the hospitalization will reduce the incidence of infection and improve the chances of success." 4. "The procedure will be performed in your room."

1. "You will begin to perform exercises to promote flexibility and reduce contractures after 5 days."

A patient with a traumatic brain injury is being evaluated for brain death. Which finding should the nurse expect in this patient? Select all that apply. 1. Absence of gag or corneal reflex 2. Absence of oculovestibular reflex 3. Apnea with PaCO2 of 66 mmHg 4. Toxic metabolic disorders 5. Response to deep stimuli

1. Absence of gag or corneal reflex 2. Absence of oculovestibular reflex 3. Apnea with PaCO2 of 66 mmHg

The nurse is caring for a patient recovering from surgery conducted in the previous 24 hours. What should the nurse do to assist this patient with pain control? 1. Administer prescribed analgesics around the clock. 2. Administer prescribed analgesics when the patient requests something for pain. 3. Assist the patient to a more comfortable position to reduce the amount of pain. 4. Offer the patient a back rub to reduce the amount of pain.

1. Administer prescribed analgesics around the clock. Rationale: Established, persistent, severe pain is more difficult to treat than pain that is at its onset. Postoperative analgesics should be administered at regular intervals around the clock to maintain a therapeutic blood level.

The nurse suspects that a patient who was in a motor vehicle crash does not have a cervical spine injury. What did the nurse assess to come to this conclusion? 1. Alert without midline cervical tenderness 2. Lacking motor response in lower extremities 3. Lacking deep tendon reflexes 4. Lethargic and confused

1. Alert without midline cervical tenderness

A patient is scheduled for total hip replacement surgery. What medication should the nurse provide to the patient prior to the surgical procedure? 1. Antibiotic 2. Antacid 3. Antiemetic 4. Anticholinergic

1. Antibiotic Rationale: Antibiotics are given preoperatively to orthopedic patients to prevent postoperative infections.

A patient with thoracic injuries sustained in a multivehicle crash is demonstrating an alteration in perfusion. What should the nurse do to help this patient? Select all that apply. 1. Auscultate lung sounds. 2. Measure blood pressure. 3. Measure central venous pressure. 4. Reduce rate of intravenous fluids. 5. Assess for jugular vein distention.

1. Auscultate lung sounds. 2. Measure blood pressure. 3. Measure central venous pressure. 5. Assess for jugular vein distention.

A victim of a multivehicle automobile crash has slurred speech and is lethargic. Which diagnostic test should be anticipated for this patient? Select all that apply. 1. Blood alcohol level 2. Urine drug screen 3. Skull x-rays 4. Chest x-ray 5. Urinalysis

1. Blood alcohol level 2. Urine drug screen

A patient was admitted with a head injury caused by rapid acceleration and deceleration. How should the nurse expect this patient's injuries to be classified? 1. Blunt 2. Shearing 3. Blast 4. Minor

1. Blunt

The nurse determines that a patient recovering from spinal anesthesia is experiencing complications from the anesthesia. Which should the nurse expect to be provided to this patient? Select all that apply. 1. Caffeine 2. Analgesics 3. Intravenous fluids 4. Epidural blood patch 5. Vasoactive medication

1. Caffeine 2. Analgesics 3. Intravenous fluids 4. Epidural blood patch Rationale: Leakage of cerebrospinal fluid (CSF) into the epidural space can cause reduced CSF pressure and postoperative headaches. Treatment may include caffeine, analgesics, hydration, and epidural blood patch.

A patient diagnosed with emphysema is being prepared for surgery. What laboratory value should the nurse review to obtain information about the patient's respiratory status? 1. Carbon dioxide 2. White blood cell count 3. Serum creatinine 4. Blood urea nitrogen

1. Carbon dioxide Rationale: The carbon dioxide level will be elevated in a patient with emphysema. This is the laboratory value that would provide information about the patient's respiratory status.

A victim of a chemical plant explosion is unconscious and brought into the emergency department. For what should the nurse assess this patient? 1. Central nervous system injury 2. Respiratory system injury 3. Pulmonary emboli 4. Radiation sickness

1. Central nervous system injury

The nurse is concerned that an older patient is at risk for a burn injury in the home. What should the nurse instruct the patient to do to reduce this risk? Select all that apply. 1. Check routinely for the odor of gas. 2. Suggest that no one smokes in the home. 3. Wear loose-fitting clothing when cooking. 4. Check the smoke detector battery annually. 5. Keep the hot water heater temperature at 120°F.

1. Check routinely for the odor of gas. 2. Suggest that no one smokes in the home. 5. Keep the hot water heater temperature at 120°F.

The nurse is assessing a patient who has returned to the care area after surgery. What should the nurse do to ensure the patient receives appropriate care? 1. Check the physician's orders to see if preoperative orders have been reordered. 2. Schedule the patient for vital signs assessments every four hours. 3. Orient the patient to person, place, and time. 4. Assess the patient's mental status.

1. Check the physician's orders to see if preoperative orders have been reordered. Rationale: The medical record needs to be checked to ensure that all orders written before surgery have been reordered after surgery, since the patient's condition has changed.

The nurse is concerned that a patient with traumatic chest injures is developing respiratory distress. What assessment finding led the nurse to come to this conclusion? Select all that apply. 1. Combative behavior 2. Absent breath sounds in left lower lobe 3. Pedal and popliteal pulses weak and irregular 4. Temperature raised 2 degrees over the last 4 hours 5. Oxygen saturation 86% on 40% oxygen face mask

1. Combative behavior 2. Absent breath sounds in left lower lobe 5. Oxygen saturation 86% on 40% oxygen face mask

The nurse is planning a program to address disaster mitigation activities in the community. What should the nurse consult when preparing this program? 1. Community disaster preparedness plan 2. Hospital administrators 3. Physicians with practices in the community 4. Schools and day care centers

1. Community disaster preparedness plan

After providing a preoperative sedative, the nurse notes that the surgical consent form has not been signed by the patient. What should the nurse do? 1. Contact the surgeon. 2. Ask the patient to sign the consent form. 3. Send the patient for surgery with an unsigned consent form. 4. Phone the operating room suite to notify the nurse that the patient has not signed the consent form.

1. Contact the surgeon. Rationale: The patient should be aware and alert before signing the consent form. The nurse should contact the surgeon in the event the patient receives preoperative sedative medication and has not yet signed the consent for surgery form. The surgeon who performs a procedure is responsible for obtaining the patient's consent for care.

An older patient, recovering from surgery, is prescribed a soft diet. Which age-related change does this type of diet support? 1. Decline in gastric motility 2. Reduced intestinal absorption 3. Lactose intolerance 4. Gall bladder insufficiency

1. Decline in gastric motility Rationale: A soft diet helps with a change in gastrointestinal functioning in the older adult.

A trauma patient is experiencing ongoing progression of shock. What finding caused the nurse to come to this conclusion? 1. Decrease in serum glucose level 2. Drop in blood urea nitrogen level 3. Increased eosinophil level 4. Low serum cardiac enzyme level

1. Decrease in serum glucose level

The nurse is evaluating laboratory values for a patient with a burn injury. What results should the nurse expect for this patient? 1. Decreased hemoglobin and elevated hematocrit levels 2. Elevated hemoglobin and elevated hematocrit levels 3. Elevated hemoglobin and decreased hematocrit levels 4. Decreased hemoglobin and decreased hematocrit levels

1. Decreased hemoglobin and elevated hematocrit levels

An older patient is being prepared for orthopedic surgery. For what potential risk should the nurse plan care? 1. Decreased tolerance of general anesthesia 2. Prolonged effects of anesthesia because of herbal supplements 3. Wound dehiscence 4. Decreased cognitive acuity

1. Decreased tolerance of general anesthesia Rationale: Older adults have age-related changes that affect physiologic, cognitive, and psychosocial responses to the stress of surgery in addition to decreased tolerance of general anesthesia and postoperative medications and delayed wound healing.

The nurse is teaching a class of older adults at a senior center about household cleaning agents that may cause burns. Which agents should be included in this discussion? Select all that apply. 1. Drain cleaners 2. Household ammonia 3. Oven cleaner 4. Toiler bowl cleaner 5. Lemon oil furniture polish

1. Drain cleaners 2. Household ammonia 3. Oven cleaner 4. Toiler bowl cleaner

A group of nurses are attending a meeting to discuss the outcome and results of care provided to victims of a recent building collapse. In which phase of the disaster planning process are the nurses participating? 1. Evaluation 2. Recovery 3. Restoration 4. Response

1. Evaluation

A patient experiences a thermal injury. Which mechanism should the nurse consider as the most likely cause of this patient's injuries? 1. Fire 2. Lightning 3. Ultraviolet radiation 4. Gunshot

1. Fire

The nurse is caring for a patient who experienced a blast injury to the eye. Which intervention would be appropriate? 1. Flush the eye with eyewash. 2. Encourage the patient to rub the eye to get out specks of dust. 3. Apply a warm compress to the eye. 4. Remove debris lodged in the eye.

1. Flush the eye with eyewash.

The nurse is reviewing the results of laboratory tests to assess the renal status of a patient who experienced a major burn event on 45% of the body 24 hours ago. Which result should the nurse expect for this patient? Select all that apply. 1. Glomerular filtration rate (GFR) reduced 2. Specific gravity elevated 3. Creatinine clearance reduced 4. BUN reduced 5. Uric acid decreased

1. Glomerular filtration rate (GFR) reduced 2. Specific gravity elevated

The nurse is presenting a program to residents of a local senior citizen housing facility about preparations in the event of a disaster. What should the nurse instruct the participants to do regarding essential preparations? 1. Have a list of medications, emergency contact numbers, and necessary survival items readily available. 2. Turn off all electrical devices before leaving the premises. 3. Pack enough nonperishable food items to last for several days. 4. Bring recreational items such as puzzles and card games for entertainment.

1. Have a list of medications, emergency contact numbers, and necessary survival items readily available.

An older patient recovering from total hip replacement surgery 8 hours ago has not been able to void spontaneously. Which actions should the nurse take to assist this patient? Select all that apply. 1. Increase fluids. 2. Turn onto the left side. 3. Palpate the bladder for distention. 4. Insert an indwelling urinary catheter. 5. Complete a bladder scan at the bedside.

1. Increase fluids. 3. Palpate the bladder for distention. 5. Complete a bladder scan at the bedside. Rationale: Promote fluid intake as allowed, monitoring intake and output. Assess for bladder distention if the patient has not voided within 7 to 8 hours after surgery. Use a portable ultrasound scanner to determine the amount of urine in the bladder.

An older patient recovering from surgery is given an antiemetic for nausea. Which manifestation indicates to the nurse that this patient is experiencing a possible reaction to the medication? 1. Involuntary muscle movements 2. Confusion 3. Dry mouth 4. Breakthrough vomiting

1. Involuntary muscle movements

An older adult patient being prepared for surgery is scheduled for an electrocardiogram. What should the nurse explain to the patient regarding the purpose of this test? 1. It is routine for all patients having general anesthesia. 2. It is used to diagnose preexisting cardiac disease. 3. It is one way to validate laboratory values 4. It is a predictor of surgical procedure success.

1. It is routine for all patients having general anesthesia. Rationale: An electrocardiogram (ECG) is ordered routinely for patients undergoing general anesthesia when they are over 40 years of age or have cardiovascular disease.

An older patient, being prepared for surgery, has a low glomerular filtration rate. Which aspect of the patient's care should the nurse realize this finding will impact? 1. Medication dosages 2. Postoperative activity level 3. Intraoperative bleeding 4. Oxygenation status

1. Medication dosages Rationale: A patient susceptible to renal insufficiency is at risk for fluid volume overload in the perioperative period and for accumulation of metabolic by-products and medications dependent on renal clearance. When this risk is known, renal function testing may be performed preoperatively. It is evaluated on the basis of glomerular filtration rate (GFR), which is estimated by using serum creatinine (reported as the eGFR) or by measuring urinary creatinine. Creatinine is a stable product of muscle mass; it is filtered by the kidneys or secreted by the kidney tubules. In kidney failure, serum creatinine rises and the GFR is low. The best indicator of GFR is the creatinine clearance, a comparison of both serum and urinary creatinine levels. Medication dosages will need to be adjusted for the older patient with a low glomerular filtration rate.

A patient has partial-thickness burns on both lower legs, over 20% of the total body surface area. How should the nurse classify this burn? 1. Moderate burn injury 2. Minor burn injury 3. Major burn injury 4. Severe burn injury

1. Moderate burn injury

A patient recovering from a major burn injury is complaining of pain. Which medication should the nurse expect to be prescribed for this patient? 1. Morphine 4 mg IV every 5 minutes 2. Morphine 10 mg IM ever 3-4 hours 3. Meperidine 75 mg IM every 3-4 hours 4. Meperidine 50 mg PO every 3-4 hours

1. Morphine 4 mg IV every 5 minutes

An organization has been alerted to a possible illness associated with the tainting of a popular over-the-counter pain reliever. For which type of event should the facility plan? 1. Nonconventional terrorist attack 2. Conventional terrorist attack 3. Accidental disaster 4. Natural disaster

1. Nonconventional terrorist attack

While the nurse is assisting a patient recovering from epidural anesthesia to ambulate, the patient becomes dizzy and has a blood pressure of 78/48 mmHg. What actions should the nurse take? Select all that apply. 1. Notify the anesthesiologist. 2. Notify the pharmacy to obtain atropine. 3. Continuously monitor blood pressure. 4. Prepare to administer intravenous fluids. 5. Prepare to administer vasoactive medications.

1. Notify the anesthesiologist. 3. Continuously monitor blood pressure. 4. Prepare to administer intravenous fluids. 5. Prepare to administer vasoactive medications. Rationale: Hypotension is common with epidural. Blood pressure should be monitored and, if critical hypotension occurs, the anesthesia provider should be alerted and expected to increase intravenous fluids and administer vasoactive medications.

A patient with multiple traumatic injuries has experienced severe blood loss and is prescribed to receive blood immediately. Because there is not enough time for type and crossmatch, which type of blood will the patient likely receive? 1. O 2. A 3. B 4. AB

1. O

A patient is diagnosed with a pneumothorax. Which type of shock is this patient at risk for developing? 1. Obstructive 2. Hypovolemic 3. Cardiogenic 4. Neurogenic

1. Obstructive

A patient recovering from surgery reports a pain level of 6 on a 0-10 pain scale but refuses additional pain medication to avoid becoming addicted. On which concept should the nurse focus when responding to this patient? 1. Physical dependence on pain medication is uncommon during the short-term postoperative use. 2. This patient already might have an addiction problem. 3. This patient might benefit from a placebo dose. 4. The physician should be notified to discuss pain management.

1. Physical dependence on pain medication is uncommon during the short-term postoperative use. Rationale: Patients might fear addiction or physical dependence on pain medications, especially opioids, postoperatively. The duration of use is typically short term, and this concern should be discussed, but is not anticipated to occur.

A patient with third-degree burns to the face learns that there will be extensive scarring once the burn heals. For which patient problem should the nurse plan interventions? 1. Powerlessness 2. Infection 3. Lack of body fluids 4. Changes in airway maintenance

1. Powerlessness

The intraoperative nurse is caring for a patient in the maintenance phase of anesthesia. Which actions should the nurse prepare to provide to the patient at this time? Select all that apply. 1. Prepare the skin. 2. Assess oxygen saturation level. 3. Participate in the surgical procedure. 4. Position the patient for the surgical procedure. 5. Measure blood pressure and heart rate.

1. Prepare the skin. 3. Participate in the surgical procedure. 4. Position the patient for the surgical procedure. Rationale: During the maintenance phase of anesthesia, the skin is prepared, the patient is positions, and surgery is performed.

A patient with a history of sleep apnea is experiencing difficulty maintaining an airway during conscious sedation. What should the nurse do to assist this patient? 1. Prepare to administer a reversal agent. 2. Begin artificial ventilations. 3. Measure oxygen saturation. 4. Apply prescribed oxygen via face mask.

1. Prepare to administer a reversal agent. Rationale: Patients with a history of sleep apnea may have difficulty with conscious sedation. The nurse should prepare to administer a reversal agent to the patient.

The nurse is reviewing the stages and phases of a disaster as part of a continuing education program in the hospital. In which order should the nurse list the stages of disaster management? 1. Preparedness, mitigation, response, recovery, and evaluation 2. Mitigation, preparedness, response, recovery, and evaluation 3. Mitigation, response, recovery, preparedness, and evaluation 4. Response, mitigation, evaluation, recovery, and preparedness

1. Preparedness, mitigation, response, recovery, and evaluation

The nurse in the same-day surgical care area is preparing a patient for surgery. What should the nurse do to ensure that this patient has a successful recovery from the surgery? 1. Provide teaching and additional resources to help the patient when at home. 2. Measure intake and output. 3. Assess vital signs. 4. Limit pain control measures since the patient will need to ambulate when leaving after the surgery.

1. Provide teaching and additional resources to help the patient when at home. Rationale: Because the postoperative phase does not end until recovery is complete, the nurse's role as educator is vital as the patient nears discharge. As the patient prepares to recuperate at home, the nurse provides information and support as needed for self-care. Written guidelines, directions, and information should accompany all aspects of teaching. Opportunities for patient and family teaching are often brief, necessitating an organized, coordinated effort.

The nurse is planning care to support the cognitive-psychosocial status for an older patient having surgery. Which intervention would be appropriate for this patient? 1. Provide time for teaching and learning. 2. Set limits with the patient. 3. Tell the patient that his physician will make all care decisions. 4. Remind the patient that the call bell is for emergencies only.

1. Provide time for teaching and learning. Rationale: To support the older patient's cognitive-psychosocial status, the nurse should provide ample time for teaching and learning.

A patient being prepared for surgery has a history of chronic obstructive pulmonary disease. Which diagnostic test may be completed prior to this patient's surgical procedure? 1. Pulmonary function tests 2. CT scan of the brain 3. Lumbar puncture 4. Abdominal MRI

1. Pulmonary function tests Rationale: Pulmonary function studies may be performed with patients who have chronic obstructive pulmonary disease to determine the extent of respiratory dysfunction.

The nurse is caring for patients when a dirty bomb detonates at a nearby shopping mall. Which types of injuries should the nurse expect to see in the victims? 1. Radiation sickness 2. Fractured limbs and spinal injury 3. Thermal burns 4. Overexertion and exhaustion

1. Radiation sickness

A patient scheduled for outpatient surgery asks why admission to the hospital is not required. What should the nurse explain as an advantage of having outpatient surgery? 1. Reduced risk of healthcare-associated infections. 2. Ability to use home care for postoperative care in the home. 3. Reduced use of postoperative medications. 4. Inadequate staffing on the surgical care areas.

1. Reduced risk of healthcare-associated infections. Rationale: Advantages to outpatient surgery include a reduced risk of healthcare-associated infections.

A patient was treated for a scald burn that caused a superficial burn over one hand and a superficial partial-thickness burn on several fingers. What should be included in this patient's teaching? Select all that apply. 1. Report any fever to the healthcare provider. 2. Report any purulent drainage to the healthcare provider. 3. Use only sterile dressings on the fingers. 4. Cleanse the areas every hour with alcohol to prevent infection. 5. Apply a topical antimicrobial agent as instructed.

1. Report any fever to the healthcare provider. 2. Report any purulent drainage to the healthcare provider. 3. Use only sterile dressings on the fingers. 5. Apply a topical antimicrobial agent as instructed.

A patient recovering from a 30% TBSA full-thickness burn has swelling and inflammation of the intact skin around the burn area. What intervention should the nurse implement to reduce this patient's risk of developing a further infection? Select all that apply. 1. Report results of blood cultures. 2. Analyze daily leukocyte counts. 3. Follow strict isolation techniques. 4. Request placement of an indwelling catheter. 5. Request a dietary consult to ensure adequate nutritional intake.

1. Report results of blood cultures. 2. Analyze daily leukocyte counts. 3. Follow strict isolation techniques. 5. Request a dietary consult to ensure adequate nutritional intake.

A patient received lorazepam (Ativan) as preoperative medication. What should the nurse assess when caring for this patient? 1. Respiratory depression 2. Nausea and vomiting 3. Confusion 4. Rash

1. Respiratory depression Rationale: The patient who received lorazepam (Ativan) should be monitored for respiratory depression, hypotension, lack of coordination, and drowsiness.

An older patient having preoperative diagnostic testing has an elevated carbon dioxide level. What should the nurse be monitoring for this patient? 1. Respiratory status and arterial blood gases 2. Serum potassium level 3. Serum sodium level 4. Intake and output

1. Respiratory status and arterial blood gases Rationale: A patient with an altered carbon dioxide level could have a history of emphysema, chronic bronchitis, asthma, pneumonia, or respiratory acidosis, or it could be caused by vomiting or nasogastric suctioning. The best nursing intervention for this patient would be to monitor the patient's respiratory status and arterial blood gases.

The nurse is planning to attend a program to learn how to respond to mass casualty incidents. Which core competencies should the nurse expect to learn during this program? Select all that apply. 1. Risk reduction, disease prevention, and health promotion 2. Nursing process 3. Policy development and planning 4. Community care 5. Ethical and legal practice

1. Risk reduction, disease prevention, and health promotion 3. Policy development and planning 4. Community care 5. Ethical and legal practice

A female patient who was raped 6 months ago seeks treatment for posttraumatic stress disorder. Which manifestation should the nurse expect to assess in this patient? 1. Severe nightmares related to the event 2. Absence of anger or shock 3. Avoiding drug and alcohol use 4. Has a very supportive family

1. Severe nightmares related to the event

A patient is admitted with severe facial injuries from a motor vehicle crash. For what should the nurse first assess this patient? 1. Signs of stridor, cough, or respiratory distress 2. Blood pressure 3. Need for suctioning 4. Loose teeth or obvious problems with the mouth

1. Signs of stridor, cough, or respiratory distress

A patient with physical injuries sustained in a gang fight has a blood pressure of 80/50 mmHg, with a pulse of 120 and thready. Which diagnostic test should the nurse expect to be performed to provide the fastest information? 1. Sonogram 2. Complete blood count 3. Urinalysis 4. Serum electrolyte levels

1. Sonogram

The nurse is preparing to discharge a patient after having outpatient surgery. Which criteria should the nurse use to determine whether the patient is eligible to be discharged? Select all that apply. 1. Stable vital signs 2. No nausea or dizziness 3. Pain controlled 4. Adequate urine output 5. Patient's expressed readiness to go home

1. Stable vital signs 2. No nausea or dizziness 3. Pain controlled 4. Adequate urine output Rationale: Following outpatient surgery, the patient will be discharged after meeting the institution's criteria, which typically include tolerance of fluids or food, stable vital signs, absence of nausea or dizziness, pain control, adequate urine output, as well as patient being oriented and demonstrating understanding of postoperative instructions.

A patient has a scald burn on the arm that is bright red, moist, and has several blisters. How should this burn be classified? Select all that apply. 1. Superficial partial-thickness burn 2. Thermal burn 3. Superficial burn 4. Deep partial-thickness burn 5. Full-thickness burn

1. Superficial partial-thickness burn 2. Thermal burn

The nurse notes an area of stasis around a patient's burn. What changes should the nurse expect to assess in this area over the next week? Select all that apply. 1. The area becomes pale. 2. The area develops necrosis. 3. The area blanches on pressure. 4. The area appears like other skin surfaces. 5. The area appears leathery and coagulated.

1. The area becomes pale. 2. The area develops necrosis. 4. The area appears like other skin surfaces.

A patient with injuries from an explosion at a nuclear power plant is confused and keeps repeating "I'm on fire." What does this information indicate to the nurse? 1. The patient may not recover. 2. The patient's clothing is burning his skin. 3. There is gastrointestinal system involvement and the patient will survive if he receives fluids. 4. There is bone marrow damage, and the patient needs oxygen for the confusion.

1. The patient may not recover.

A patient has a burn that is pale, waxy, and with large flat blisters. What should the nurse respond when asked about the severity and healing time of the injury? 1. The wound is a deep partial-thickness burn and will take more than 3 weeks to heal. 2. The wound is a superficial partial-thickness burn and could take up to 2 weeks to heal. 3. The wound is a superficial burn and will take up to 3 weeks to heal. 4. The wound is a full-thickness burn and will take 1 to 2 weeks to heal.

1. The wound is a deep partial-thickness burn and will take more than 3 weeks to heal.

When monitoring the vital signs of a patient who has sustained a major burn injury, the nurse assesses a heart rate of 112. What should the nurse determine about this finding? 1. This heart rate is normal for the patient's post-burn injury condition. 2. The patient is demonstrating manifestations consistent with the onset of an infection. 3. The patient is demonstrating manifestations consistent with an electrolyte imbalance. 4. The patient is demonstrating manifestations consistent with renal failure.

1. This heart rate is normal for the patient's post-burn injury condition.

Victims of a school bus crash are being triaged at the accident site. What principle should be followed when implementing reverse triage? Select all that apply. 1. Used when there is a mass casualty event with more than 100 victims. 2. Works on the principle of the greatest good for the greatest number. 3. Categorizes victims needing the most support and emergency care as red, so they can be treated first. 4. Color-codes victims most likely to survive as black to be treated first. 5. Works on the principle of the greatest good for the most critically ill.

1. Used when there is a mass casualty event with more than 100 victims. 2. Works on the principle of the greatest good for the greatest number.

During a support group session, a client says, "My husband hit me a lot, but when he threatened to start hitting our kids, I stabbed him. No jury will believe me because my husband can lie to anyone and be believed." If no one in the group responds, which statement is the therapeutic response by the nurse? 1."Abuse is a horribly difficult thing to experience. Can anyone in the group relate to what she's feeling?" 2."Yes. Everyone here was ill-used and abused, but what makes you think that this is a reason to stab someone?" 3."Everyone agrees that you couldn't let him hurt your children. But is there anything you would do differently?" 4."Your story is very much like every woman's here. The problem is getting a jury to see that you were justified in stabbing him."

1."Abuse is a horribly difficult thing to experience. Can anyone in the group relate to what she's feeling?"

A 60-kg client has sustained third-degree burns over 40% of the body. Using the Parkland (Baxter) formula, the minimum fluid requirements are which during the first 24 hours after the burn? 1.9600 mL of lactated Ringer's solution 2.4800 mL of 0.9% normal saline solution 3.2400 mL of 0.45% normal saline solution 4.1200 mL of 5% dextrose in water solution

1.9600 mL of lactated Ringer's solution Rationale: The Parkland (Baxter) formula is 4 mL of lactated Ringer's solution × kg body weight × percent burn. The calculation is performed as follows: 4 mL × 60 kg × 40 = 9600 mL. ALTHOUGH no math was need because burn victims receive lactated ringer & #1 is the only option using LR

A child is seen in the school nurse's office with complaints of pain in his right forearm. In reviewing the child's record the nurse notes that he has a history of being physically abused by the mother. Which should be the initial intervention with this child? 1.Assess the child's physical status. 2.Ask the child how the injury occurred. 3.Report the case as suspected child abuse. 4.Observe the interactions between the child and his friends.

1.Assess the child's physical status.

The nurse in the hospital emergency department is notified by emergency medical services that several victims who survived a plane crash will be transported to the hospital. Victims are suffering from cold exposure because the plane plummeted and was submerged in a local river. What is the initial action of the nurse? 1.Call the nursing supervisor to activate the agency disaster plan. 2.Supply the triage rooms with bottles of sterile water and normal saline. 3.Call the intensive care unit to request that nurses be sent to the emergency department. 4.Call the laundry department and ask the department to send as many warm blankets as possible to the emergency depa

1.Call the nursing supervisor to activate the agency disaster plan.

The community health nurse is providing a teaching session to firefighters in a small community regarding care of a burn victim at the scene of injury. The nurse instructs the firefighters that in the event of a tar burn, which is the immediate action? 1.Cooling the injury with water 2.Removing all clothing immediately 3.Removing the tar from the burn injury 4.Leaving any clothing that is saturated with tar in place

1.Cooling the injury with water Rationale: Scald burns and tar or asphalt burns are treated by immediate cooling with saline solution or water, if available, or immediate removal of the saturated clothing. Clothing that is burned into the skin is not removed because increased tissue damage and bleeding may result. No attempt is made to remove tar from the skin at the scene.

What health effects best describe a client who is the victim of abuse or negligence? Select all that apply. 1.Depression 2.Chronic fatigue 3.Involuntary shaking 4.Motivation to persevere 5.Interrupted sleeping patterns

1.Depression 2.Chronic fatigue 3.Involuntary shaking 5.Interrupted sleeping patterns

The community health nurse is preparing to teach personnel and family preparedness for disasters to a group of parents of school-age children. Which items should the nurse plan to include in disaster preparedness? Select all that apply. 1.Flashlight 2.Supply of batteries 3.Battery-operated radio 4.Extra pair of eyeglasses 5.4-week supply of water 6.4-week supply of nonperishable food

1.Flashlight 2.Supply of batteries 3.Battery-operated radio 4.Extra pair of eyeglasses

A client taking calcium carbonate chewable tablets and ranitidine is on nothing by mouth (NPO) status and has a nasogastric (NG) tube in place after suffering bilateral burns to the legs. The nurse determines that the client's gastrointestinal (GI) status is least satisfactory if which finding is noted on assessment? 1.Gastric pH of 3 2.Absence of abdominal discomfort 3.GI drainage that is guaiac negative 4.Presence of hypoactive bowel sounds

1.Gastric pH of 3 Rationale: The gastric pH should be maintained at 7 or greater with the use of prescribed antacids and histamine 2 (H2) receptor-blocking agents. Lowered pH (to the acidic range) in the absence of food or tube feedings can lead to erosion of the gastric lining and ulcer development. Absence of discomfort and bleeding (guaiac-negative drainage) are normal findings. The client's bowel sounds may be expected to be hypoactive in the absence of oral or NG tube intake.

The nurse is reviewing the laboratory test results for a client admitted to the burn unit 3 hours after an explosion that occurred at a worksite. The client has a severe burn injury that covers 35% of the total body surface area (TBSA). The nurse is most likely to note which finding on the laboratory report? 1.Hematocrit 60% (0.60) 2.Serum albumin 4.8 g/dL (48 g/L) 3.Serum sodium 144 mEq/L (144 mmol/L) 4.White blood cell (WBC) count 9000 mm3 (9 × 109/L)

1.Hematocrit 60% (0.60) Rationale: Extensive burns greater than 25% of the TBSA result in generalized body edema in both burned and unburned tissues and a decrease in circulating intravascular blood volume. Hematocrit levels are elevated in the first 24 hours after injury as a result of hemoconcentration from the loss of intravascular fluid. The normal hematocrit level ranges from 37% to 47% (0.37 to 0.47) (female) and 42% to 52% (0.42 to 0.52) (male). The remaining options identify normal laboratory values.

The industrial nurse is providing instructions to a group of employees regarding care to a client in the event of a chemical burn injury. The nurse instructs the employees that which is the firstconsideration in immediate care? 1.Removing all clothing, including gloves, shoes, and any undergarments 2.Determining the antidote for the chemical and placing the antidote on the burn site 3.Leaving all clothing in place until the client is brought to the emergency department 4.Lavaging the skin with water and avoiding brushing powdered chemicals off the clothing

1.Removing all clothing, including gloves, shoes, and any undergarments Rationale: In a chemical burn injury, the burning process continues as long as the chemical is in contact with the skin. All clothing, including gloves, shoes, and undergarments, is removed immediately, and water lavage is instituted before and during transport to the emergency department. Powdered chemicals are first brushed off the client before lavage is performed.

The nurse is preparing to care for a burn client scheduled for an escharotomy procedure being performed for a third-degree circumferential arm burn. The nurse understands that which finding is the anticipated therapeutic outcome of the escharotomy? 1.Return of distal pulses 2.Brisk bleeding from the site 3.Decreasing edema formation 4.Formation of granulation tissue

1.Return of distal pulses Rationale: Escharotomies are performed to relieve the compartment syndrome that can occur when edema forms under nondistensible eschar in a circumferential third-degree burn. The escharotomy releases the tourniquet-like compression around the arm. Escharotomies are performed through avascular eschar to subcutaneous fat. Although bleeding may occur from the site, it is considered a complication rather than an anticipated therapeutic outcome. Usually, direct pressure with a bulky dressing and elevation control the bleeding, but occasionally an artery is damaged and may require ligation. Escharotomy does not affect the formation of edema. Formation of granulation tissue is not the intent of an escharotomy.

A client is diagnosed with stage I Lyme disease, and the nurse assesses the client for disease manifestations. Which should the nurse expect to note as the hallmark characteristic of this stage? 1.Skin rash 2.Arthralgias 3.Neurological deficits 4.Enlarged and inflamed join

1.Skin rash

A patient is admitted with trauma to the integumentary system. Which type of skin trauma should the nurse prepare to assess? Select all that apply. 1. Cutaneous 2. Abrasion 3. Laceration 4. Contusion 5. Keloid

2. Abrasion 3. Laceration 4. Contusion

The nurse is preparing to assist during a surgical procedure. For which reason should the surgical scrub be completed? Select all that apply. 1. Sterilize the skin. 2. Apply an antimicrobial residue on the skin. 3. Prevent the need to wear surgical gloves during the procedure. 4. Remove dirt, skin oils, and transient microorganisms from the skin. 5. Improve patient safety by removing the number of organisms on personnel.

2. Apply an antimicrobial residue on the skin. 4. Remove dirt, skin oils, and transient microorganisms from the skin. 5. Improve patient safety by removing the number of organisms on personnel. Rationale: The purpose of a surgical scrub is to improve patient safety by removing the number of organisms on personnel, remove dirt, skin oils, and transient microorganisms from the skin, and leave an antimicrobial residue on the skin to inhibit the growth of microbes for several hours,

The nurse notes that a large number of community members with diabetes have experienced insufficient glucose control and cold symptoms since a tornado ripped through the area a few weeks ago. What should the nurse consider as the cause for these symptoms? Select all that apply. 1. Blood stasis in extremities 2. Hindered immune response 3. Change in basal metabolic rate 4. Paralysis of gastrointestinal tract 5. Alteration in nutrient metabolism

2. Hindered immune response 3. Change in basal metabolic rate 5. Alteration in nutrient metabolism

The administrator of a multihospital healthcare system contacts the National Response Framework (NRF) for help with managing care for victims of a flood. What assistance should the administrator expect to receive from this organization? Select all that apply. 1. Standing orders used when caring for victims of a flood 2. Information about local relief agencies available to assist 3. Frequency in which the area has flooded over the last 10 years 4. Amount of time a flood victim can withstand submersion in cold water 5. Names of organizations within the private sector that can help the victims

2. Information about local relief agencies available to assist 5. Names of organizations within the private sector that can help the victims

The community is holding a memorial service to honor members whose lives were lost in a flood one year ago. In which stage of the disaster recovery process is this community? 1. Restoration 2. Recovery 3. Mitigation 4. Evaluation

2. Recovery

The nurse is planning the coding for a triage disaster plan. Which colors should the nurse use for this plan? Select all that apply. 1. White 2. Red 3. Yellow 4. Black 5. Green

2. Red 3. Yellow 4. Black 5. Green

A client presents at the primary health care provider's office with complaints of a ring-like rash on his upper leg. Which question should the nurse ask first? 1."Do you have any cats in your home?" 2."Have you been camping in the last month?" 3."Have you or close contacts had any flu-like symptoms within the last few weeks?" 4."Have you been in physical contact with anyone who has the same type of rash?"

2."Have you been camping in the last month?"

The nursing educator has just completed a lecture to a group of nurses regarding care of the client with a burn injury. A major aspect of the lecture was care of the client at the scene of a fire. Which statement, if made by a nurse, indicates a need for further instruction? 1."Flames should be doused with water." 2."The client should be maintained in a standing position." 3."Flames may be extinguished by rolling the client on the ground." 4."Flames may be smothered by the use of a blanket or another cover."

2."The client should be maintained in a standing position." Rationale: The client should be placed or maintained in a supine position; otherwise, flames may spread to other parts of the body, causing more extensive injury. Flame burns may be extinguished by rolling the client on the ground, smothering the flames with a blanket or other cover, or dousing the flames with water.

A client suspected of having stage I Lyme disease is seen in the health care clinic and is told that the Lyme disease test result is positive. The client asks the nurse about the treatment for the disease. In responding to the client, the nurse anticipates that which intervention will be part of the treatment plan? 1.Ultraviolet light therapy 2.A 14- to 21-day course of doxycycline 3.No treatment unless symptoms develop 4.Treatment with intravenous (IV) penicillin G

2.A 14- to 21-day course of doxycycline

The nurse in charge of a nursing unit is asked to select the hospitalized clients who can be discharged so that hospital beds can be made available for victims of a community disaster. Select the clients who can be safely discharged. Select all that apply. 1.A client with dyspnea 2.A client experiencing sinus rhythm 3.A client receiving oral anticoagulants 4.A client with chronic atrial fibrillation 5.A client experiencing third-degree heart block 6.A client who has not voided since before surgery

2.A client experiencing sinus rhythm 3.A client receiving oral anticoagulants 4.A client with chronic atrial fibrillation

The nurse in charge of a nursing unit is asked to select the hospitalized clients who can be discharged so that hospital beds can be made available for victims of a community disaster. Which clients can be safely discharged? Select all that apply. 1.A client with chest pain 2.A client with a Holter monitor 3.A client receiving oral antibiotics 4.A client experiencing sinus rhythm 5.A client newly diagnosed with atrial fibrillation 6.A client experiencing third-degree heart block who requires a pacemaker

2.A client with a Holter monitor 3.A client receiving oral antibiotics 4.A client experiencing sinus rhythm

The nurse is the first responder after a tornado has destroyed many homes in the community. Which victim should the nurse attend to first? 1.A pregnant woman who exclaims, "My baby is not moving." 2.A woman who is complaining, "My leg is bleeding so bad, I am afraid it is going to fall off!" 3.A young child standing next to an adult family member who is screaming, "I want my mommy!" 4.An older victim who is sitting next to her husband sobbing, "My husband is dead. My husband is dead."

2.A woman who is complaining, "My leg is bleeding so bad, I am afraid it is going to fall off!"

A client is brought to the emergency department with partial-thickness burns to his face, neck, arms, and chest after trying to put out a car fire. The nurse should implement which nursing actions for this client? Select all that apply. 1.Restrict fluids. 2.Assess for airway patency. 3.Administer oxygen as prescribed. 4.Place a cooling blanket on the client. 5.Elevate extremities if no fractures are present. 6.Prepare to give oral pain medication as prescribed.

2.Assess for airway patency. 3.Administer oxygen as prescribed. 5.Elevate extremities if no fractures are present. Rationale: The primary goal for a burn injury is to maintain a patent airway, administer intravenous (IV) fluids to prevent hypovolemic shock, and preserve vital organ functioning. Therefore, the priority actions are to assess for airway patency and maintain a patent airway. The nurse then prepares to administer oxygen. Oxygen is necessary to perfuse vital tissues and organs. An IV line should be obtained and fluid resuscitation started. The extremities are elevated to assist in preventing shock and decrease fluid moving to the extremities, especially in the burn-injured upper extremities. The client is kept warm, because the loss of skin integrity causes heat loss. The client is placed on NPO (nothing by mouth) status because of the altered gastrointestinal function that occurs as a result of a burn injury.

A client is brought to the emergency room with a snake bite to the arm. Which treatment interventions should the nurse anticipate? Select all that apply. 1.Apply ice to the site. 2.Deliver supplemental oxygen. 3.Apply a tourniquet just above the site. 4.Maintain the extremity at the level of the heart. 5.Infuse crystalloid fluids through 2 large-bore intravenous (IV) lines. 6.Immobilize the affected extremity in a position of function with a splint.

2.Deliver supplemental oxygen. 4.Maintain the extremity at the level of the heart. 5.Infuse crystalloid fluids through 2 large-bore intravenous (IV) lines. 6.Immobilize the affected extremity in a position of function with a splint.

The nurse is caring for a client with a severe burn who is scheduled for an autograft to be placed on the lower extremity. The nurse creates a postoperative plan of care for the client and should include which intervention in the plan? 1.Maintain the client in a prone position. 2.Elevate and immobilize the grafted extremity. 3.Maintain the grafted extremity in a flat position. 4.Keep the grafted extremity covered with a blanket.

2.Elevate and immobilize the grafted extremity. Rationale: Autografts placed over joints or on lower extremities are elevated and immobilized after surgery for 3 to 7 days, depending on the surgeon's preference. This period of immobilization allows the autograft time to adhere and attach to the wound bed, and the elevation minimizes edema. Keeping the client in a prone position and covering the extremity with a blanket can disrupt the graft site.

A client is admitted to the hospital emergency department after receiving a burn injury in a house fire. The skin on the client's trunk is tan, dry, and hard. It is edematous but not very painful. The nurse determines that this client's burn should be classified as which type? 1.Superficial 2.Full-thickness 3.Deep partial-thickness 4.Partial-thickness superficial

2.Full-thickness Rationale: Full-thickness burns involve the epidermis, the full dermis, and some of the subcutaneous fat layer. The burn appears to be a tan or fawn color, with skin that is hard, dry, and inelastic. Edema is severe, and the accumulated fluid compresses tissue underneath because of eschar formation. Some nerve endings have been damaged, and the area may be insensitive to touch, with little or no pain.

The nurse is evaluating fluid resuscitation attempts in the burn client. Which finding indicates adequate fluid resuscitation? 1.Disorientation to time only 2.Heart rate of 95 beats/minute 3.+1 palpable peripheral pulses 4.Urine output of 30 mL over the past 2 hours

2.Heart rate of 95 beats/minute Rationale: When fluid resuscitation is adequate, the heart rate should be less than 120 beats/minute, as indicated in option 2. In addition, adequacy of fluid volume resuscitation can be evaluated by determining if urine output is at least 30 mL/hour, peripheral pulses are +2 or better, and the client is oriented to client, place, and time.

The nurse is planning care for a client who suffered a burn injury and has a negative self-image related to keloid formation at the burn site. The keloid formation is indicative of which condition? 1.Nerve damage 2.Hypertrophy of collagen fibers 3.Compromised circulation at the burn site 4.Increase in subcutaneous tissue at the burn site

2.Hypertrophy of collagen fibers Rationale: Keloids are visible as excessive scar formation and result from hypertrophy of collagen fibers. Nerves conduct sensory and motor impulses from the skin. The vasculature provides blood vessels with nourishment and assists in thermoregulation. Subcutaneous tissue provides for heat insulation, mechanical shock absorption, and caloric reserve.

The nurse is planning care for a client returning from the operating room after having an autograft applied to the right lower extremity. Which nursing intervention is focused on promoting graft "take"? 1.Monitor temperature every 4 hours. 2.Leave the dressing intact for 3 to 5 days. 3.Apply an ice pack to the site to decrease edema formation. 4.Maintain the right lower extremity in a dependent position.

2.Leave the dressing intact for 3 to 5 days.

A client is seen in the ambulatory care clinic for a superficial burn to the arm. On assessing the skin at the burn injury, what will the nurse observe? 1.White color 2.Pink or red color 3.Weeping blisters 4.Insensitivity to pain and cold

2.Pink or red color Rationale: Superficial burns are pink or red without any blistering. The skin blanches to touch, may be edematous and painful, and heals on its own, usually within 1 week. A white color characterizes deep partial-thickness burns. Weeping blisters characterize partial-thickness superficial burns. Deep full-thickness burns are associated with insensitivity to pain and cold.

The nurse is reviewing the manual of disaster preparedness and response for the annual hospital disaster drill. The nurse reads that which are functions of the American Red Cross (ARC) as opposed to the Federal Emergency Management Agency (FEMA) in the United States? Select all that apply. 1.Provide monetary relief. 2.Provide crisis counseling. 3.Identify and train personnel. 4.Issue presidential declarations. 5.Deploy National Guard troops. 6.Handle inquiries from families.

2.Provide crisis counseling. 3.Identify and train personnel. 6.Handle inquiries from families.

The nurse in the emergency department is caring for a young female victim of sexual assault. The client's physical assessment is complete, and physical evidence has been collected. The nurse notes that the client is withdrawn, confused, and at times physically immobile. How should the nurse interpret these behaviors? 1.Signs of depression 2.Reactions to a devastating event 3.Evidence that the client is a high suicide risk 4.Indicative of the need for hospital admission

2.Reactions to a devastating event

A client arrives at the health care clinic and tells the nurse that she was just bitten by a tick and would like to be tested for Lyme disease. The client tells the nurse that she removed the tick and flushed it down the toilet. Which actions are most appropriate? Select all that apply. 1.Tell the client that testing is not necessary unless arthralgia develops. 2.Tell the client to avoid any woody, grassy areas that may contain ticks. 3.Instruct the client to immediately start to take the antibiotics that are prescribed. 4.Inform the client to plan to have a blood test 4 to 6 weeks after a bite to detect the presence of the disease. 5.Tell the client that if this happens again, to never remove the tick but vigorously scrub the area with an antiseptic.

2.Tell the client to avoid any woody, grassy areas that may contain ticks. 3.Instruct the client to immediately start to take the antibiotics that are prescribed. 4.Inform the client to plan to have a blood test 4 to 6 weeks after a bite to detect the presence of the disease.

The nurse in charge of a nursing unit is asked to select those hospitalized clients who can be discharged so that hospital beds can be made available for victims of a community disaster. Which clients can be safely discharged? Select all that apply. 1.The client with heart failure (HF) who has bilateral rhonchi 2.The client who 24 hours earlier gave birth to her second child by caesarean delivery 3.The 48-hour postoperative client who has undergone an ileostomy because of ulcerative colitis 4.The client with peritonitis caused by a ruptured appendix who is febrile with a temperature of 102º F (38.9º C) 5.The 2-day postoperative client who has undergone total knee replacement and is ambulating with a walker 6.The 3-day postoperative client who has undergone coronary artery bypass grafting and is ready for rehabilitation

2.The client who 24 hours earlier gave birth to her second child by caesarean delivery 3.The 48-hour postoperative client who has undergone an ileostomy because of ulcerative colitis 5.The 2-day postoperative client who has undergone total knee replacement and is ambulating with a walker 6.The 3-day postoperative client who has undergone coronary artery bypass grafting and is ready for rehabilitation

The nurse is administering fluids intravenously as prescribed to a client who sustained superficial partial-thickness burn injuries of the back and legs. In evaluating the adequacy of fluid resuscitation, the nurse understands that which assessment would provide the most reliable indicator for determining the adequacy? 1.Vital signs 2.Urine output 3.Mental status 4.Peripheral pulses

2.Urine output Rationale: Successful or adequate fluid resuscitation in the client is signaled by stable vital signs, adequate urine output, palpable peripheral pulses, and clear sensorium. However, the most reliable indicator for determining adequacy of fluid resuscitation, especially in a client with burns, is the urine output. For an adult, the hourly urine volume should be 30 to 50 mL.

The health education nurse provides instructions to a group of clients regarding measures that will assist in preventing skin cancer. Which instructions should the nurse provide? Select all that apply. 1.Sunscreen should be applied every 8 hours. 2.Use sunscreen when participating in outdoor activities. 3.Wear a hat, opaque clothing, and sunglasses when in the sun. 4.Avoid sun exposure in the late afternoon and early evening hours. 5.Examine your body monthly for any lesions that may be suspicious.

2.Use sunscreen when participating in outdoor activities. 3.Wear a hat, opaque clothing, and sunglasses when in the sun. 5.Examine your body monthly for any lesions that may be suspicious. Rationale: The client should be instructed to avoid sun exposure between the hours of brightest sunlight: 10 a.m. and 4 p.m. Sunscreen, a hat, opaque clothing, and sunglasses should be worn for outdoor activities. The client should be instructed to examine the body monthly for the appearance of any cancerous or precancerous lesions. Sunscreen should be applied 30 minutes to 1 hour before sun exposure, and reapplied every 2 to 3 hours, and after swimming or sweating; otherwise, the duration of protection is reduced.

An adult client trapped in a burning house has suffered burns to the back of the head, the upper half of the posterior trunk, and the back of both arms. Using the rule of nines, what percentage does the nurse determine the extent of the burn injury to be? Fill in the blank.

22.5% Rationale: According to the rule of nines, the posterior side of the head equals 4.5%, the back of both arms equals 9%, and the upper half of the posterior trunk equals 9%, totaling 22.5%.

The emergency response team is setting up an area in which to triage victims of a building blast in a major metropolitan area. Which zone should the nurse identify to set up the location for triage? 1. Hot 2. Warm 3. Cold 4. Clean

3. Cold

An adolescent is experiencing anaphylactic shock after being stung by a swarm of bees. Which medication should the nurse anticipate providing to this patient? Select all that apply. 1. Diuretics 2. Antibiotics 3. Epinephrine 4. Beta2-agonists 5. Antihistamine

3. Epinephrine 4. Beta2-agonists 5. Antihistamine

A patient is receiving emergent care for burns over 45% of TBSA. What should the nurse realize is occurring during the inflammation phase of these injuries? Select all that apply. 1. Epithelial cells cover the wound. 2. Granulation tissue begins to form. 3. Fibrin is deposited within the damaged tissue. 4. Aggregation of platelets within the damaged tissue. 5. Hemostasis walls off the wound from systemic circulation.

3. Fibrin is deposited within the damaged tissue. 4. Aggregation of platelets within the damaged tissue. 5. Hemostasis walls off the wound from systemic circulation.

A trauma patient is being assessed with the Trauma and Injury Severity Score (TRISS) system. What assessment data should the nurse use with this scoring system? Select all that apply. 1. Diastolic blood pressure 2. Heart rate 3. Glasgow coma scale 4. Systolic blood pressure 5. Respiratory rate

3. Glasgow coma scale 4. Systolic blood pressure 5. Respiratory rate

A patient is coded red after a mass casualty accident. What care should the nurse provide to this patient?' 1. Send the patient home. 2. Evaluate the patient in the next few hours. 3. Immediately evaluate this patient. 4. Permit the family to be with the patient as death approaches.

3. Immediately evaluate this patient.

A subway derailed injuring approximately 250 passengers. For which type of situation should the emergency department manager prepare? 1. Natural disaster 2. Multiple-casualty incident 3. Mass-casualty incident 4. Accidental disaster

3. Mass-casualty incident

The National Weather Service has announced the likelihood of a large snow event in a major metropolitan area. For which type of health problem should the nurses in the emergency department prepare? 1. Stress-related injuries 2. Crushing injuries 3. Myocardial infarctions 4. Burns

3. Myocardial infarctions

Survivors of a bioterrorist attack are receiving basic care and support in a local motel until further plans for placement can be made. Several of these victims have been receiving care for autoimmune disorders. What should the nurse consider when providing these victims with food over the next several hours? Select all that apply. 1. Offer fresh salads. 2. Ensure that fruit is fresh. 3. Provide bottled water. 4. Provide cooked or processed meals. 5. Fill pitchers with tap water.

3. Provide bottled water. 4. Provide cooked or processed meals.

The nurse is preparing a group of Cub Scouts for an overnight camping trip and instructs the Scouts about the methods to prevent Lyme disease. Which statement by one of the Scouts indicates a need for further instruction? 1."I need to bring a hat to wear during the trip." 2."I should wear long-sleeved tops and long pants." 3."I should not use insect repellents because it will attract the ticks." 4."I need to wear closed shoes and socks that can be pulled up over my pants."

3."I should not use insect repellents because it will attract the ticks."

A female victim of a sexual assault is being seen in the crisis center. The client states that she still feels "as though the rape just happened yesterday," even though it has been a few months since the incident. Which is the most appropriate nursing response? 1."You need to try to be realistic. The rape did not just occur." 2."It will take some time to get over these feelings about your rape." 3."Tell me more about the incident that causes you to feel like the rape just occurred." 4."What do you think that you can do to alleviate some of your fears about being raped again?"

3."Tell me more about the incident that causes you to feel like the rape just occurred."

An adult client was burned in an explosion. The burn initially affected the client's entire face (anterior half of the head) and the upper half of the anterior torso, and there were circumferential burns to the lower half of both arms. The client's clothes caught on fire, and the client ran, causing subsequent burn injuries to the posterior surface of the head and the upper half of the posterior torso. Using the rule of nines, what would be the extent of the burn injury? 1.18% 2.24% 3.36% 4.48%

3.36% Rationale: According to the rule of nines, with the initial burn, the anterior half of the head equals 4.5%, the upper half of the anterior torso equals 9%, and the lower half of both arms equals 9%. The subsequent burn included the posterior half of the head, equaling 4.5%, and the upper half of the posterior torso, equaling 9%. This totals 36%.

A client is suspected of having stage I Lyme disease. The nurse anticipates that which will be part of the treatment plan for the client? 1.Daily oatmeal baths for 2 weeks 2.No treatment unless symptoms develop 3.A 14- to 21-day course of oral antibiotic therapy 4.Treatment with intravenously administered antibiotics

3.A 14- to 21-day course of oral antibiotic therapy

The nurse is the first responder at the scene of a train accident. Which victim should the nurse attend to first? 1.A victim experiencing excruciating pain 2.A victim experiencing moderate anxiety 3.A victim experiencing airway obstruction 4.A victim experiencing altered level of consciousness

3.A victim experiencing airway obstruction

The nurse is providing an educational session to new employees, and the topic is abuse of the older client. The nurse helps the employees identify which client as most typically a victim of abuse? 1.A man who has moderate hypertension 2.A man who has newly diagnosed cataracts 3.A woman who has advanced Parkinson's disease 4.A woman who has early diagnosed Lyme disease

3.A woman who has advanced Parkinson's disease

The nurse is performing an assessment on a client who sustained circumferential burns of both legs. Which assessment would be the initial priority in caring for this client? 1.Assessing heart rate 2.Assessing respiratory rate 3.Assessing peripheral pulses 4.Assessing blood pressure (BP)

3.Assessing peripheral pulses Rationale: The client who receives circumferential burns to the extremities is at risk for altered peripheral circulation. The priority assessment would be to assess for peripheral pulses to ensure that adequate circulation is present. Although the respiratory rate and BP also would be assessed, the priority with a circumferential burn is assessment for the presence of peripheral pulses because the airway is not affected in this case.

The nurse employed in a hospital is waiting to receive a report from the laboratory via the facsimile (fax) machine. The fax machine activates and the nurse expects the report, but instead receives a sexually oriented photograph. Which is the most appropriate initial nursing action? 1.Call the police. 2.Cut up the photograph and throw it away. 3.Call the nursing supervisor and report the occurrence. 4.Call the laboratory and ask for the name of the individual who sent the photograph.

3.Call the nursing supervisor and report the occurrence.

The nurse is caring for a client following an autograft and grafting to a burn wound on the right knee. What would the nurse anticipate to be prescribed for the client? 1.Out-of-bed activities 2.Bathroom privileges 3.Immobilization of the affected leg 4.Placing the affected leg in a dependent position

3.Immobilization of the affected leg Rationale: Autografts placed over joints or on the lower extremities after surgery often are elevated and immobilized for 3 to 7 days. This period of immobilization allows the autograft time to adhere to the wound bed. Getting out of bed, going to the bathroom, and placing the grafted leg dependent would put stress on the grafted wound.

A client is undergoing fluid replacement after being burned on 20% of her body 12 hours ago. The nursing assessment reveals a blood pressure of 90/50 mm Hg, a pulse rate of 110 beats/minute, and a urine output of 20 mL over the past hour. The nurse reports the findings to the primary health care provider (PHCP) and anticipates which prescription? 1.Transfusing 1 unit of packed red blood cells 2.Administering a diuretic to increase urine output 3.Increasing the amount of intravenous (IV) lactated Ringer's solution administered per hour 4.Changing the IV lactated Ringer's solution to one that contains 5% dextrose in water

3.Increasing the amount of intravenous (IV) lactated Ringer's solution administered per hour Rationale: Fluid management during the first 24 hours following a burn injury generally includes the infusion of (usually) lactated Ringer's solution. Lactated Ringer's solution is an isotonic solution that contains electrolytes that will maintain fluid volume in the circulation. Fluid resuscitation is determined by urine output, and hourly urine output should be at least 30 mL/hour. The client's urine output is indicative of insufficient fluid resuscitation, which places the client at risk for inadequate perfusion of the brain, heart, kidneys, and other body organs. Therefore, the PHCP would prescribe an increase in the amount of IV lactated Ringer's solution administered per hour. There is nothing in the situation that calls for blood replacement, which is not used for fluid therapy for burn injuries. Administering a diuretic would not correct the problem because fluid replacement is needed. Diuretics promote the removal of the circulating volume, thereby further compromising the inadequate tissue perfusion. Intravenous 5% dextrose solution is isotonic before administered but is hypotonic once the dextrose is metabolized. Hypotonic solutions are not appropriate for fluid resuscitation of a client with significant burn injuries.

A client sustained a burn from cutaneous exposure to lye. At the site of injury, copious irrigation to the site was performed for 1 hour. On admission to the hospital emergency department, the nurse assesses the burn site. Which findings would indicate that the chemical burn process is continuing? 1.Eschar 2.Intact blisters 3.Liquefaction necrosis 4.Cherry-red, firm tissue

3.Liquefaction necrosis Rationale: Alkalis, such as lye, cause a liquefaction necrosis, and exposure to fat results in formation of a soapy coagulum. Thick, leathery eschar forms with exposure to acids or heat. Intact blisters indicate a partial-thickness thermal injury. Cherry-red, firm tissue can occur as a result of thermal injury.

The nurse manager is observing a new nursing graduate caring for a burn client in protective isolation. The nurse manager intervenes if the new nursing graduate planned to implement which unsafe component of protective isolation technique? 1.Using sterile sheets and linens 2.Performing strict hand-washing technique 3.Wearing gloves and a gown only when giving direct care to the client 4.Wearing protective garb, including a mask, gloves, cap, shoe covers, gowns, and plastic apron

3.Wearing gloves and a gown only when giving direct care to the client Rationale: In protective isolation, the nurse needs to protect the client at all times from any potential infectious contact. Thorough hand washing should be done before and after each contact with the burn-injured client. Sterile sheets and linens are used because of the client's high risk for infection. Protective garb, including gloves, cap, masks, shoe covers, gowns, and plastic apron, need to be worn when in the client's room and when directly caring for the client.

A burn client has been having 1% silver sulfadiazine applied to burns twice a day for the past 3 days. Which laboratory abnormality indicates that the client is experiencing a side or adverse effect of this medication? 1.Serum sodium of 120 mEq/L (120 mmol/L) 2.Serum potassium of 3.0 mEq/L (3.0 mmol/L) 3.White blood cell count of 3000 mm3 (3 × 109/L) 4.pH of 7.30, PaCO2 of 32 mm Hg (32 mmHg), HCO3- of 19 mEq/L (19 mmol/L)

3.White blood cell count of 3000 mm3 (3 × 109/L) Rationale: Transient leukopenia typically occurs after 2 to 3 days of treatment. Knowing this and knowing normal white blood cell values will direct you to option 3. Although options 1, 2, and 4 are abnormal findings, they are not associated with this medication.

The nursing staff is planning to attend a presentation on disaster prevention. What should the nurse expect the purpose of this education to be? 1. Learn the organization's disaster plan 2. Participate in the plan for handling a disaster 3. Educate on how to recognize possible terrorists 4. Learn how to participate in mitigation

4. Learn how to participate in mitigation

An older woman is brought to the emergency department for treatment of a fractured arm. On physical assessment, the nurse notes old and new ecchymotic areas on the client's chest and legs and asks the client how the bruises were sustained. The client, although reluctant, tells the nurse in confidence that her son frequently hits her if supper is not prepared on time when he arrives home from work. Which is the most appropriate nursing response? 1."Oh, really? I will discuss this situation with your son." 2."Let's talk about the ways you can manage your time to prevent this from happening." 3."Do you have any friends who can help you out until you resolve these important issues with your son?" 4."As a nurse, I am legally bound to report abuse. I will stay with you while you give the report and help find a safe place for you to stay."

4."As a nurse, I am legally bound to report abuse. I will stay with you while you give the report and help find a safe place for you to stay."

Collagenase is prescribed for a client with a severe burn to the hand. The home care nurse provides instructions to the client regarding the use of the medication. Which client statement indicates an accurate understanding of the use of this medication? 1."I will apply the ointment at bedtime and in the morning." 2."I will apply the ointment once a day and leave it open to the air." 3."I will apply the ointment twice a day and leave it open to the air." 4."I will apply the ointment once a day and cover it with a sterile dressing."

4."I will apply the ointment once a day and cover it with a sterile dressing." Rationale: Collagenase is used to promote debridement of dermal lesions and severe burns. It is applied once daily and covered with a sterile dressing. The remaining options are incorrect.

Silver sulfadiazine is prescribed for a client with a partial-thickness burn, and the nurse provides teaching about the medication. Which statement made by the client indicates a need for further teaching about the treatments? 1."The medication is an antibacterial." 2."The medication will help heal the burn." 3."The medication should be applied directly to the wound." 4."The medication is likely to cause stinging every time it is applied."

4."The medication is likely to cause stinging every time it is applied." Rationale: Silver sulfadiazine is an antibacterial that has a broad spectrum of activity against gram-negative bacteria, gram-positive bacteria, and yeast. It is applied directly to the wound to assist in healing. It does not cause stinging when applied.

A client arrives at the emergency department following a burn injury that occurred in the basement at home, and an inhalation injury is suspected. What would the nurse anticipate to be prescribed for the client? 1.100% oxygen via an aerosol mask 2.Oxygen via nasal cannula at 6 L/minute 3.Oxygen via nasal cannula at 15 L/minute 4.100% oxygen via a tight-fitting, nonrebreather face mask

4.100% oxygen via a tight-fitting, nonrebreather face mask Rationale: If an inhalation injury is suspected, administration of 100% oxygen via a tight-fitting nonrebreather face mask is prescribed until carboxyhemoglobin levels fall (usually below 15%). In inhalation injuries, the oropharynx is inspected for evidence of erythema, blisters, or ulcerations. The need for endotracheal intubation also is assessed. Administration of oxygen by aerosol mask and cannula are incorrect and would not provide the necessary oxygen supply needed for adequate tissue perfusion for the client with a likely inhalation injury.

Assessment and diagnostic evaluation reveal that a client seen in the ambulatory care clinic has stage II Lyme disease. The clinic nurse identifies which assessment finding as most characteristic of this stage? 1.Arthralgias 2.Joint enlargement 3.Erythematous rash 4.Cardiac conduction deficits

4.Cardiac conduction deficits

The nurse is caring for a client who sustained superficial partial-thickness burns on the anterior lower legs and anterior thorax. Which finding does the nurse expect to note during the resuscitation/emergent phase of the burn injury? 1.Decreased heart rate 2.Increased urinary output 3.Increased blood pressure 4.Elevated hematocrit levels

4.Elevated hematocrit levels Rationale: The resuscitation/emergent phase begins at the time of injury and ends with the restoration of capillary permeability, usually at 48 to 72 hours following the injury. During the resuscitation/emergent phase, the hematocrit level increases to above normal because of hemoconcentration from the large fluid shifts. Hematocrit levels of 50% to 55% (0.50 to 0.55) are expected during the first 24 hours after injury, with return to normal by 36 hours after injury. Initially, blood is shunted away from the kidneys and renal perfusion and glomerular filtration are decreased, resulting in low urine output. The burn client is prone to hypovolemia, and the body attempts to compensate by increased pulse rate and lowered blood pressure. Pulse rates are typically higher than normal, and the blood pressure is decreased as a result of the large fluid shifts.

Mafenide acetate is prescribed for a client with a burn injury to the hand. Which should the nurse include in the instructions to the client regarding the use of this medication? 1.If stinging occurs, discontinue the medication. 2.Apply a thinner film than prescribed to the burn site if the medication stings. 3.If local stinging and burning occur after the medication is applied, notify the health care provider. 4.It is normal to experience local discomfort and stinging and burning after the medication is applied.

4.It is normal to experience local discomfort and stinging and burning after the medication is applied. Rationale: Mafenide acetate is bacteriostatic for both gram-negative and gram-positive organisms and is used to treat second- and third-degree burns to reduce the number of bacteria present in avascular tissues. The client should be informed that the medication will cause local discomfort, stinging, and burning and that this is normal. Therefore, options 1, 2, and 3 are incorrect.

A client who is being evaluated for thermal burn injuries to the arms and legs complains of thirst and asks the nurse for a drink. Which action by the nurse is most appropriate? 1.Allow the client to have full liquids. 2.Give the client small glasses of clear liquids. 3.Order the client a full meal tray with extra liquids. 4.Keep the client on NPO (nothing by mouth) status.

4.Keep the client on NPO (nothing by mouth) status. Rationale: The client should be maintained on NPO status because burn injuries frequently result in paralytic ileus. The client also should be told that fluids could cause vomiting because of the effect of the burn injury on gastrointestinal tract functioning. Mouth care should be given as appropriate to alleviate the sensation of thirst.

An elderly client is being seen at the primary health care provider's (PHCP's) office. The client is thin and has multiple bruises on his exposed skin, and the nurse suspects elder neglect and abuse. His caregiver does all the talking and updates the nurse about the client since last being seen. The nurse should take which bestaction? 1.Ask the client if he feels safe in the home. 2.Ask the caregiver specific questions about the elder's health problems. 3.Educate the caregiver to increase the client's dietary fat intake to help increase his weight. 4.Recognize that neglect and abuse is common in this population and try to follow up with the client privately.

4.Recognize that neglect and abuse is common in this population and try to follow up with the client privately.

The visiting nurse observes that the older male client is confined by his daughter-in-law to his room. When the nurse suggests that he walk to the den and join the family, he says, "I'm in everyone's way; my daughter-in-law needs me to stay here." Which is the most important action for the nurse to take? 1.Say to the daughter-in-law, "Confining your father-in-law to his room is inhumane." 2.Suggest to the client and daughter-in-law that they consider a nursing home for the client. 3.Say nothing, because it is best for the nurse to remain neutral and wait to be asked for help. 4.Suggest appropriate resources to the client and daughter-in-law, such as respite care and a senior citizens center.

4.Suggest appropriate resources to the client and daughter-in-law, such as respite care and a senior citizens center.

The nurse is caring for a client who was admitted to the burn unit after sustaining a burn injury covering 30% of the body. What is the most appropriate time frame for the emergent phase? 1.The entire period of time during which rehabilitation occurs 2.The period from the time the client is stable to the time when all burns are covered with skin 3.The period from the time the burn was incurred to the time when the client is admitted to the hospital 4.The period from the time the burn was incurred to the time when the client is considered physiologically stable

4.The period from the time the burn was incurred to the time when the client is considered physiologically stable Rationale: The emergent phase of burn care generally extends from the time the burn injury is incurred until the time when the client is considered physiologically stable. The acute phase lasts until all full-thickness burns are covered with skin. The rehabilitation period lasts approximately 5 years for an adult and includes reintegration into society.

The nurse is monitoring the fluid balance of a client with a burn injury. The nurse determines that the client is less than adequately hydrated if which information is noted during assessment? 1.Urine pH of 6 2.Urine that is pale yellow 3.Urine output of 40 mL/hr 4.Urine specific gravity of 1.032

4.Urine specific gravity of 1.032 Rationale: The client who is not adequately hydrated will have an elevated urine specific gravity. Normal values for urine specific gravity range from approximately 1.005 to 1.030. Pale yellow urine is a normal finding, as is a urine output of 40 mL/hr (minimum is 30 mL/hr). A urine pH of 6 is adequate (4.6 to 8.0 normal), and this value is not used in monitoring hydration status.

Silver sulfadiazine is prescribed for a client with a burn injury. Which laboratory finding requires the need for follow-up by the nurse? 1.Glucose level of 99 mg/dL (5.65 mmol/L) 2.Platelet level of 300,000 mm3 (300 × 109/L) 3.Magnesium level of 1.5 mEq/L (0.75 mmol/L) 4.White blood cell count of 3000 mm3 (3.0 × 109/L)

4.White blood cell count of 3000 mm3 (3.0 × 109/L) Rationale: Silver sulfadiazine is used for the treatment of burn injuries. Adverse effects of this medication include rash and itching, blue-green or gray skin discoloration, leukopenia, and interstitial nephritis. The nurse should monitor a complete blood count, particularly the white blood cells, frequently for the client taking this medication. If leukopenia develops, the primary health care provider is notified and the medication is usually discontinued. The white blood cell count noted in option 4 is indicative of leukopenia. The other laboratory values are not specific to this medication and are also within normal limits.


Kaugnay na mga set ng pag-aaral

Lecture 3 Learning Outcomes - BIOL2085 Cell Biology - Professor Maria Torres

View Set

Ch 29: Nonmalignant Hematologic Disorders

View Set

HESI Cardiovascular defect- PEDS

View Set

Abeka 5th grade History, Quiz 12

View Set